Download as pdf or txt
Download as pdf or txt
You are on page 1of 59

Polity Level-2 Question Bank Historical Background, Constituent Assembly, Salient Features

Note:
2. Residual powers with the Centre — Canadian
Constitution
#1: This PDF is intended solely for personal use. We
kindly ask you to avoid unauthorised sharing and
3. Supreme Court's advisory jurisdiction — French
support our affordable initiatives. If you are aware
Constitution
of anyone who might benefit from these efforts,
please explore our Refer and Earn programme. For
4. Freedom of trade, commerce and intercourse
those who know at least 25 individuals who could
— Japanese Constitution
benefit from our initiatives, we encourage you to
consider our partner programme. Our referral and
How many pairs given above are correctly
partner benefits are among the best in the industry.
matched?
#2: In 2023 Prelims, a significant number of (a) Only one
multiple-choice questions introduced varied (b) Only two
response options (e.g., any 1, any 2, only 1, only 2, all
(c) Only three
three, etc.). To facilitate easy identification during
revision, these specific questions in our question (d) All four
bank are highlighted using a combination of bold
and italics for their options. 3. Consider the following statements with
reference to the Indian Independence Act of 1947:
#3: To report inaccuracies, if any, please email us at
content@meritnest.com. 1. The Act made the Constituent Assembly of India
a fully sovereign body.
#4: You can check corrections to this PDF from the
page you downloaded it from (click on relevant 2. It made the Constituent Assembly the first
errata link). Parliament of free India.

#5: We will also communicate all corrections via 3. The office of the Governor-General of India was
email ~1 month prior to the upcoming Prelims to abolished by the Act.
everyone who downloaded this PDF from
https://www.meritnest.com How many statements given above are correct?
(a) Only one
1. With reference to legal philosophy of the (b) Only two
Constitution, consider the following statements: (c) All three
(d) None
1. ‘Modernism’ demands that the constitution
should be interpreted as it was understood at the
4. Consider the following pairs of features
time when it was ratified.
borrowed and the sources, with reference to the
Constitution of India:
2. ‘Originalism’ demands that the constitution oslo
should be updated with times to encompass
1. Concurrent List — Canadian Constitution
changing societal needs.
2. Method of President's election — Irish
Which of the above statement(s) is/are correct?
Constitution
(a) 1 only 9654469135

(b) 2 only 3. Procedure established by law — Australian


Constitution
katya124905@gmail.com
(c) Both 1 and 2
(d) Neither 1 nor 2 4. Procedure to amend the Constitution — South
African Constitution
2. Consider the following pairs of features of the
Indian Constitution and the countries from which How many of the pairs given above are correct?
they were borrowed:
(a) Only one
1. Cabinet system — US Constitution (b) Only two

https://www.meritnest.com Page 1
Polity Level-2 Question Bank Historical Background, Constituent Assembly, Salient Features

(c) Only three


Select the correct answer using the code given
(d) All four below:
(a) 1 and 2 only
5. Which of the following explains the reason oslo
behind inclusion of emergency powers in the (b) 2 and 3 only katya124905@gmail.com
Indian constitution? (c) 1 and 3 only
(d) 1, 2 and 3
1. To safeguard federal character of constitution
8. Consider the following statements:
2. To safeguard unity and integrity of country
1. Daman and Diu were acquired by India from the
3. To safeguard Fundamental rights of citizens
Portuguese by police action.
4. To safeguard democratic political system
2. Sikkim was added to India through referendum.
Select the correct answer using the code given
3. Puducherry was administered by India as an
below:
‘acquired territory’ till 1962.
(a) 1, 2, 3 and 4
(b) 2 and 4 only How many statements given above are correct?
(c) 1, 2 and 3 only (a) Only one
(d) 2, 3 and 4 only (b) Only two
(c) All three
6. Which of the following provisions of the Indian (d) None
Constitution shows that India is a Secular State?
9. Consider the following statements:
1. The State shall endeavour to secure for all the
citizens a Uniform Civil Code.
1. The power of Judicial review of the Supreme
Court in India is narrower than that of the Supreme
2. Religious instructions shall be provided in
Court in the United States.
educational institutions maintained by the State.
2. The American constitution provides for
3. Religious denomination shall have the right to
procedure established by law against that of due
manage its own religious affairs.
process of law provided by the Indian constitution.
9654469135
4. Citizens shall not be discriminated on the
Which of the above Statement(s) is/are correct?
grounds of religion in respect of employment under
the State. (a) 1 only
(b) 2 only
Select the correct answer using the code given
(c) Both 1 and 2
below:
(d) Neither 1 nor 2
(a) 1, 2 and 3 only
(b) 3 and 4 only 10. The constitutional position of British
(c) 1, 3 and 4 only territories in India was explicitly defined in which
(d) 1 and 4 only of the following Act?
(a) The Charter Act of 1813
7. Which of the following features ensures (b) The Regulating Act of 1773
constitutionalism in India?
(c) The Pitts India Act of 1784
1. Rule of law (d) The Charter Act of 1833

2. Separation of power 11. Which of the following statements are


incorrect regarding Ninth Schedule of the
3. Judicial review Constitution?

https://www.meritnest.com Page 2
Polity Level-2 Question Bank Historical Background, Constituent Assembly, Salient Features

(d) 1 and 3
1. It was added to the constitution by the first
Amendment.
14. Which of the following provisions of the
2. Currently the laws placed under it are outside the Government of India Act of 1919, were similar to
purview of judicial review. the actual practices adopted by the Constitution
of independent India with some changes?
3. Acts of the State legislatures cannot be put under
it by the Parliament. 1. Direct elections to the central legislative councils.

Select the correct answer using the code given 2. Division of powers in Federal, Provincial and the
below: Concurrent List.

(a) 1 and 2 only 3. Bicameral legislature.


(b) 2 and 3 only
(c) 1 and 3 only 4. Dyarchy at the Central government.
(d) 1, 2 and 3
Select the correct answer using the code given
below:
12. With reference to the Government of India Act
of 1935, consider the following statements: (a) 1 and 3 only
(b) 2 and 4 only
1. The Act provided for establishment of India as a (c) 1, 2 and 3 only
federation of the British provinces and princely
9654469135 oslo
(d) 3 and 4 only
states.

2. The Act introduced bicameralism in the British 15. With reference to Judiciary in India and United
provinces. States of America, consider the following
statements:
3. The Act designated the Governor-General of
India as a nominal head of the State. 1. Unlike Indian Supreme Court, the Original
Jurisdiction of American Supreme Court is confined
How many statements given above are correct? to Centre- state disputes only.

(a) Only one 2. Unlike the Supreme Court of India, the American
(b) Only two Supreme Court does not have advisory jurisdiction.
(c) All three
3. Unlike American Supreme Court, the appellate
(d) None
jurisdiction of Indian Supreme Court is confined to
constitutional cases only.
13. Which of the following statements is/are
correct regarding the Charter Act 1853? 4. Unlike Indian Supreme Court, American Supreme
Court has a wide discretion to grant special leave to
1. It introduced local representation in the Indian appeal in any matter.
Legislative Council for the first time.
How many of the above given statements are not
2. For the first time, it authorized the Local correct?
Governments in India to impose taxes on persons.
(a) Only one
3. The Act provided for an open competition system (b) Only two katya124905@gmail.com
of selection and recruitment of civil servants. (c) Only three
(d) All four
Select the correct answer using the code given
below:
16. With reference to the role of constitution/
(a) 1 only government in promoting equality among Indian
(b) 2 and 3 citizens, which of the following statements is
(c) 1 and 2 incorrect?

https://www.meritnest.com Page 3
Polity Level-2 Question Bank Historical Background, Constituent Assembly, Salient Features

(a) The constitution allows for differentiating 4. Larger dependence on Conventions


between people to ensure equity.
(b) The constitution has given more preference to 5. Single Citizenship
Equality in comparison to liberty.
How many statements given above are correct?
(c) The government can violate Article 14 of the
Constitution for implementing some of the (a) Only two
Directive Principles of State Policy. (b) Only three
(d) Special assistance in the form of affirmative (c) Only four
action helps in promoting equality.
(d) All five

17. Which of the following functions were


20. Which of the following features of the
performed by the Constituent Assembly of India?
Constitution of India are borrowed from the
Constitution of the United States?
1. Ratified India’s Commonwealth membership.
1. Fundamental rights.
2. Elected first President of India.
2. Judicial review.
3. Appointed India’s first Prime Minister till the
formation of new parliament in 1952.
3. Ideal of justice in the Preamble.
Select the correct answer using the code given
4. Federation with a strong Centre.
below: 9654469135 oslo

(a) 1 only Select the correct answer using the code given
(b) 1 and 2 only below:
(c) 2 and 3 only (a) 1 and 2 only
(d) 1, 2 and 3 (b) 3 and 4 only
(c) 1, 2 and 3 only
18. With reference to the history — of (d) 1, 2, 3 and 4
constitutional legislation in Modern India, what
was meant by the term Communal Award?
21. Indian brand of socialism is called as
(a) Offer of the British to create different states for ‘Democratic Socialism’.
all different religious communities at the time of
independence. Which one of the following options best describe
(b) Award announced by the British to recognise ‘democratic socialism’ of India?
efforts of individuals to reduce communal tensions (a) It involves nationalisation of all means of
during non- cooperation movement. production and distribution and abolition of private
(c) Privileges in the form of large share in properties.
government services awarded by the British to (b) It follows the ideals of market economy and
communities who were loyal to them. pursues the path of globalisation towards economic
(d) A separate electorate awarded to members of integration of domestic markets.
different communities after the Second Round (c) It holds faith in a ‘mixed economy’ where both
Table Conference. public and private sectors co-exist side by side.
(d) It tends to lean heavily towards public sector to
19. Which of the following is/are salient features create economic equality within the capitalist
katya124905@gmail.com
of the Constitution of India? setup.

1. Three-tier Governance model


22. With reference to ‘Source of Indian
Constitution', consider the following pairs:
2. Judicial Supremacy over parliamentary
sovereignty
1. Irish Constitution — Fundamental Duties and the
ideal of justice.
3. Blend of Rigidity and Flexibility

https://www.meritnest.com Page 4
Polity Level-2 Question Bank Historical Background, Constituent Assembly, Salient Features

2. Canadian Constitution — Suspending 25. Indian Constitution is considered as one of the


Fundamental Rights during Emergency. lengthiest in the world, which one of the following
is not the reason for it?
3. Australian Constitution — Joint sitting of the two
(a) A major part of it is based on the Government of
Houses of Parliament.
India Act, 1935, which was a bulky document.
4. Soviet Constitution — Directive Principles of (b) It accommodates provisions related to
State Policy. government mechanisms of both centre and states.
(c) Fundamental principles of governance and
How many of the above given pairs are correctly matters of administration are given in great detail.
matched? (d) Procedures related to preparation of electoral
(a) Only one rolls and registration of political parties are
included in the Constitution.
(b) Only two
(c) Only three
26. The “Instrument of Instructions” contained in
(d) All four the Government of India Act 1935 have been
incorporated in the Constitution of India in the
23. With reference to the Communal Award, year 1950 as:
consider the following statements:
(a) Fundamental Rights
1. It was introduced on the recommendations of (b) Directive Principles of State Policy
Simon Commission. (c) Extent of executive power of State
(d) Conduct of business of the Government of India
2. It introduced separate electorates for Sikhs,
Indian Christians, and Anglo-Indians for the first
27. Which of the following offices is/are included
time.
in the both second and third schedule of the
Constitution?
3. It led to the signing of Poona Pact which gave
reserved seats to the depressed classes.
1. The Judges of the High Courts
How many statements given above are correct?
2. The Judges of the Supreme Court
(a) Only one
(b) Only two 3. The Comptroller and Auditor-General of India
(c) All three
4. The Prime Minister's Office
(d) None katya124905@gmail.com
Select the correct answer using the code given
24. Consider the following statements
oslo with regard below:
to the idea of liberty:
(a) 1 and 2 only
1. Negative Liberty is concerned with the (b) 1, 2 and 3 only
recognition of a minimum area in which the (c) 2, 3 and 4 only
individual can act unobstructed by others. (d) 1, 2, 3 and 4 9654469135

2. Positive Liberty recognises that an individual can


28. With reference to the Government of India Act
be free only in society and not outside of it.
of 1935, consider the following statements:
Which of the statements given above is/are
1. It provided for the adoption of the dyarchy at the
correct?
Centre.
(a) 1 only
(b) 2 only 2. It abolished the Council of India, established by
the Government of India Act of 1858.
(c) Both 1 and 2
(d) Neither 1 nor 2 3. It provided for the separate electorates for
depressed classes, women and labour.

https://www.meritnest.com Page 5
Polity Level-2 Question Bank Historical Background, Constituent Assembly, Salient Features

Indian Constitution:
How many statements given above are correct?
(a) Only one 1. Both the second and third schedules contains
(b) Only two provision related to Vice President.
(c) All three
2. The President and the Governor of states are
(d) None included only in Second Schedule and not in the
Third Schedule.
29. Which of the following factors make the Indian
Constitution more acceptable to the people at 3. The Comptroller and Auditor General of India is
large? included in both the Schedules.

1. National leaders proposed the subjection of 4. Both the Schedules can be amended by a simple
constitution to referendum of people. majority of Parliament.
katya124905@gmail.com

2. It was supported by popular leaders who enjoyed How many of the above given statements are
immense credibility. correct?
(a) Only one
3. Constitution was framed by representatives from
all sections of Indian society. (b) Only two
(c) Only three
Select the correct answer using the code given (d) All four
below:
oslo
(a) 1 and 2 only 32. With reference to ‘Constituent Assembly;
(b) 2 and 3 only consider the following statements:
9654469135
(c) 1 and 3 only
1. The assembly was empowered to alter any law
(d) 1, 2 and 3 made by the British Parliament in relation to India.

30. Match the following pairs: 2. The members of the Muslim League from the
Indian Dominion never entered the Assembly.
Feature borrowed:
3. Both H.C. Mukherjee and V.T. Krishnamachari
1. Office of the Governor
were elected as the Vice- Presidents of the
2. Supreme Court's advisory jurisdiction Assembly.
3. Joint sitting of the two Houses of Parliament
4. Dr Sachchidanand Sinha acted as the chairperson
4. Election to the Rajya Sabha of the assembly whenever it met as the legislative
body.
Source
How many of the above given statements are
A. Canadian Constitution
correct?
B. South African Constitution
(a) Only one
C. Australian Constitution (b) Only two
D. Government of India Act, 1935 (c) Only three
(d) All four
Choose the correct answer using the code below:
(a) 1-A, 2-C, 3-B, 4-D 33. The Constituent Assembly appointed a number
(b) 1-D, 2-A, 3-C, 4-B of committees to deal with different tasks of
constitution-making. In this context, consider the
(c) 1-B, 2-D, 3-A, 4-C
following committees:
(d) 1-C, 2-B, 3-D, 4-A
1. Rules of Procedure Committee
31. Consider the following statements regarding
the Second Schedule and the Third Schedule of 2. Steering Committee

https://www.meritnest.com Page 6
Polity Level-2 Question Bank Historical Background, Constituent Assembly, Salient Features

(d) None
3. Finance and Staff Committee
36. Consider the following statements:
4. Ad-hoc Committee on the National Flag
1. The Constituent Assembly of India was formed
How many of the above given committees were
under the scheme formulated by the Cripps
headed by Dr. Rajendra Prasad?
Mission.
(a) Only one
(b) Only two 2. The Constituent assembly generally followed the
policy of consensus and accommodation with
(c) Only three
respect to drafting of the Constitution.
(d) All four
Which of the statements given above is/are
34. Consider the following pairs correct? katya124905@gmail.com

(a) 1 only
Schedules of the Constitution — Subject matter
(b) 2 only
1. Third Schedule — Allocation of seats in the Rajya (c) Both 1 and 2
Sabha to the states. (d) Neither 1 nor 2

2. Fifth Schedule — Provisions relating to the


37. Consider the following pairs of Schedules of
administration of tribal areas like in Meghalaya.
the Constitution of India and its subject matter:
oslo
3. Eighth Schedule — Division of powers between
1. Second Schedule – provisions for emoluments,
the Union and the States.
allowances, privileges
9654469135
4. Twelfth Schedule — Powers, authority and
2. Fourth Schedule — forms of Oaths or
responsibilities of Panchayats.
Affirmations
How many pairs given above are incorrectly
3. Seventh Schedule — Division of powers between
matched?
the Union and the States
(a) Only one
(b) Only two 4. Eleventh Schedule — Powers, authority and
responsibilities of Municipalities
(c) Only three
(d) All four How many pairs given above are correctly
matched?
35. Consider the following statements:
(a) Only one
1. The Act dropped the title of Emperor of India (b) Only two
from the royal titles of the King of England. (c) Only three
(d) All four
2. The Act ended the British rule in India and
declared India as an independent and sovereign
38. With reference to ‘Indian Council Act of 1861’,
state.
consider the following statements:
3. The Act provided for the appointment of some
1. It restored the legislative powers to the Bombay
civil servants by the Secretary of State for India.
and Madras Presidencies.
How many of the above given statements are
2. It established a 15-member Council of India to
correct about the Indian Independence Act of
assist the secretary of state for India.
1947?
(a) Only one 3. It ended the system of double government by
(b) Only two abolishing the Board of Control and Court of
Directors.
(c) All three

https://www.meritnest.com Page 7
Polity Level-2 Question Bank Historical Background, Constituent Assembly, Salient Features

4. It empowered the Viceroy to issue ordinances 1. Jawaharlal Nehru — Home Affairs


without the concurrence of the legislative council.
2. Sardar Patel — Information and Broadcasting
How many of the above given statements are
correct? 3. Rajkumari Amrit Kaur — Health
(a) Only one
4. Dr. Rajendra Prasad — Education
(b) Only two
(c) Only three How many of the above given pairs is/are correctly
(d) All four matched?
(a) Only one
39. With reference to the constitution, consider (b) Only two
the following statements:
(c) Only three
1. Constitutions are required only in democratic (d) All four
countries.
43. Consider the following statements:
2. Constitution is a legal document that does not
deal with ideals and values. 1. Charter Act of 1813 ended all the commercial
activities of the East India Company.
3. A constitution gives its citizens a moral identity.
2. Charter Act of 1833 created Government of India
How many statements given above are correct? having authority over the entire territorial area
possessed by the British in India.
(a) Only one
(b) Only two 3. Charter Act of 1833 separated the legislative and
(c) All three executive functions of the Governor-General’s
(d) None council.

How many statements given above are correct?


40. Which of the following statements with
reference to the Constituent Assembly of newly (a) Only one
independent India is correct? (b) Only two
(a) B R Ambedkar was the Chief Draftsman of (c) All three
Constitution in the Assembly. (d) None
katya124905@gmail.com
(b) None of the members of the Assembly were
Women. 44. With reference to the Indian and British
oslo as its
(c) The Assembly adopted the Elephant Parliamentary System, consider the following
symbol/seal. statements:
(d) S Varadachari was the legal advisor to the
Assembly. 1. Both the Indian and British Parliamentary
systems are based on Doctrine of the Sovereignty
9654469135
of Parliament.
41. The demand for Constituent Assembly was
accepted for the first time by the British
2. In both the systems, the Prime Minister must be
Government through:
from the lower house of Parliament.
(a) Wavell plan
(b) Simon Commission 3. Britain has the system of legal responsibility of
the ministers, while India has no such system.
(c) Gandhi Irwin Pact
(d) August Offer 4. Both the systems have an elected Head of the
State.
42. Consider the following pairs (Member —
Portfolio) with reference to the First Cabinet of How many of the above given statements are
Independent India (1947): correct?

https://www.meritnest.com Page 8
Polity Level-2 Question Bank Historical Background, Constituent Assembly, Salient Features

(a) Only one (b) Ordinary Legislative power of the Parliament is


(b) Only two superior to its Constituent power.
(c) All three (c) Amendments to the Constitution can be made
both through a special procedure and through the
(d) None
manner in which ordinary laws are enacted.
(d) Private members cannot introduce
45. Which of the following are the valid criticisms
constitutional amendment bill.
of constituent assembly of India?

1. Members are not directly elected by people 48. With reference to the composition of India’s
Constituent Assembly, consider the following
2. Nonfunctional as a legislative body statements:

3. Lack of representation to legal professionals 1. It was constituted on the basis of


recommendations of the Cabinet Mission.
4. Dominated by single political party
2. Members from the Princely States were drawn
Select the correct answer using the code given through nomination.
below:
3. Seats were reserved for various minorities but
(a) 1 only there was no separate electorate.
(b) 1 and 4 only
(c) 1, 2 and 3 only How many of the above given statements are
correct?
(d) 2, 3 and 4 only
(a) Only one
46. Which of the following are the reasons for the (b) Only two
inclusion of emergency provisions in the Indian (c) All three
constitution?
(d) None
1. To safeguard the sovereignty, unity, integrity and
security of the country. 49. With reference to Indian form of secularism,
consider the following statements:
2. To vest wide powers to centre so as to handle
special situations. 1. Indian secularism deals only with religious
freedom of individual and not of community.
3. To prevent maladministration or corruption in
katya124905@gmail.com
the country. 2. Indian secularism allows the state-supported
religious reform.
oslo
4. To sort out intra-party problems of the ruling
party. 3. Indian secularism focuses on both inter and intra
religious equality.
Select the correct answer using the code given
below: 4. Under Indian secularism the state cannot aid any
religious institution.
9654469135
(a) 1 and 2 only
(b) 1, 2 and 3 only How many statements given above are correct?
(c) 1, 3 and 4 only (a) Only one
(d) 2 and 3 only (b) Only two
(c) Only three
47. Which one of the following is a reason why
(d) All four
Indian Constitution is described as the blend of
rigidity and flexibility?
50. With reference to the Constitution and its
(a) Both the Central and the state governments can framework, consider the following statements:
initiate the amendment to the Constitution.

https://www.meritnest.com Page 9
Polity Level-2 Question Bank Historical Background, Constituent Assembly, Salient Features

1. A written, or unwritten constitution needs to (c) 1-3-2-4


exist tor a country to be called a nation. (d) 3-4-1-2

2. A constitution specifies who has the power to


53. The Supreme court of India can declare certain
make decisions in a society.
laws/executive actions as unconstitutional under
which of the following conditions?
3. A constitution provides a set of basic rules that
allow for minimal coordination amongst members
1. If the law enacted by parliament violates a
of a society.
provision of the constitution.
4. A constitution provides an identity to its citizens.
2. Any laws that are against the federal provisions
of the constitution.
How many of the above given statements are
correct?
3. Any legislative/executive actions that
(a) Only one contravenes the Directive Principles of State Policy
(b) Only two (DPSP).
(c) Only three
Select the correct answer using the code given
(d) All four below:
(a) 1 and 2 only
51. Consider the following statements regarding
the term ‘Asymmetric Federalism’: (b) 2 and 3 only
(c) 1 only
1. In Asymmetric Federalism, a particular state can (d) 1 and 3 only
have greater executive powers than other states.
54. Consider the following statements:
2. It overlooks the diverse cultural, linguistic, and
historical differences among constituent units.
1. Indian parliament is a sovereign body similar to
British parliament.
3. Indian Constitution provides for the Asymmetric
federalism.
2. Indian state has an elected head while the Britain
state has hereditary head.
How many of the above given statements are
correct?
3. Political sovereignty in India is with the people
(a) Only one and not in a single individual.
(b) Only two
How many statements given above are correct?
(c) All three
(d) None (a) Only one
(b) Only two
oslo
52. Arrange the following events in the (c) All three
chronological order of their occurrence: (d) None

1. Adoption of National Anthem


55. Consider the following statements:
9654469135
2. Ratification of Commonwealth membership
1. Communist Party of India did not win any seats in
the elections to the Constituent Assembly of India.
katya124905@gmail.com
3. Adoption of National Flag in its present form
2. Unionist Party won the second highest number of
4. Adoption of National Emblem
seats, after Indian National Congress, in the
elections to the Constituent Assembly of India.
Select the correct answer using the codes given
below:
Which of the statements given above is/are
(a) 1-2-4-3 correct?
(b) 3-2-1-4 (a) 1 only

https://www.meritnest.com Page 10
Polity Level-2 Question Bank Historical Background, Constituent Assembly, Salient Features

(b) 2 only 59. Consider the following statements with


reference to Acts and Commissions during the
(c) Both 1 and 2
British rule:
(d) Neither 1 nor 2
1. Indian Councils Act of 1833 provided for
56. With reference to the Charter Act of 1813, nomination of Indians as non-official members of
consider the following statements: legislative council.

1. It granted permission to the Christian 2. India Council Act of 1892 enlarged the functions
missionaries to come to India and engage in of the Legislative Councils and gave them powers to
religious proselytisation. discuss the Budget.

2. It extended the trade monopoly of the company 3. The Simon Commission recommended the
in India for another ten years. extension of responsible Government in the
katya124905@gmail.com
provinces.
3. It set aside Rs.1 Lakh every year on the education
of Indians. How many statements given above are correct?
(a) Only one
How many statements given above are correct?
(b) Only two
(a) Only one
(c) All three
(b) Only two
(d) None
(c) All three
oslo
(d) None 60. With reference to ‘Composition of Constituent
Assembly of India, consider the following
57. With reference to the ‘Objectives
9654469135Resolution, statements:
introduced in the first session of the Constituent
Assembly, consider the following statements: 1. Each British Indian province and princely state
were allotted fixed one seat in the constituent
1. It declared India as an Independent Sovereign assembly.
Republic.
2. The seats allocated to each British province were
2. It provided adequate safeguards for minorities to be divided among the Muslims, Sikhs and
and depressed classes. General.

3. It guaranteed to people of India, the equality of 3. The members from chief commissioners’
status of opportunity. Provinces were represented in the constituent
assembly.
How many of the above given statements are
correct? How many of the above given statements are
correct?
(a) Only one
(b) Only two (a) Only one
(c) All three (b) Only two
(d) None (c) All three
(d) None
58. In the federation established by the
Government of India Act of 1935 residuary Power 61. Which of the following features of Indian
was given to the: Constitution is/are borrowed from Australian
Constitution?
(a) Federal Legislature
(b) Governor-General 1. Concurrent List
(c) Provincial Legislature
(d) Provincial Governors 2. Procedure for amendment of the Constitution

3. Federation with a strong Centre

https://www.meritnest.com Page 11
Polity Level-2 Question Bank Historical Background, Constituent Assembly, Salient Features

(d) 1, 2, 3, 4, 5 and 6
4. Joint sitting of the two Houses of Parliament
64. Which of the following elements were
5. Freedom of trade and commerce introduced for the first time in British India by the
oslo
Government of India Act,katya124905@gmail.com
1919?
Select the correct answer using the codes given
below: 1. Dyarchy at provinces
(a) 1, 2 and 4 only
2. Bicameralism
(b) 1, 4 and 5 only
(c) 2, 3 and 5 only 3. Communal representation in legislature
(d) 2, 3, 4 and 5 only
4. Association of Indians with the Viceroy’s
62. Which among the following constitutional executive council
provision form the basis of India being a welfare
state? 5. Allowing Provincial Councils to determine on
women’s voting rights
1. Fundamental Rights
Select the correct answer using the code given
2. Directive Principles of State Policy below:
(a) 1 and 2 only
3. Fundamental Duties
(b) 1, 3 and 4 only
4. Preamble (c) 1, 2 and 5 only
(d) 2, 3, 4 and 5 only
Select the correct answer using the codes given
below. 65. Which of the following statements is/are
(a) 1, 2 and 4 only correct about the concept of ‘Police State’?
(b) 1 and 4 only
1. It puts forward that the state can perform various
(c) 1, 2 and 3 only functions of social welfare.
(d) 2 and 3 only
2. It is concerned with the maintenance of law and
63. Provisions related to which of the following order and defence of the country against external
topics of the Constitution of India came into force aggression.
9654469135
on 26th November 1949?
3. Laissez faire is the fundamental economic system
1. Provisional Parliament and policy necessarily followed by all Police states.

2. Citizenship Select the correct answer using the code given


below:
3. Directive Principles of State Policy (a) 1 and 2 only
(b) 1 and 3 only
4. Elections
(c) 2 only
5. Planning Commission (d) 1, 2 and 3

6. Fundamental Rights 66. Why Indian judiciary has been described as an


integrated judiciary system?
Select the correct answer using the codes given
below: (a) High court enforces only state laws of concerned
respective states.
(a) 1, 2, 3 and 5 only
(b) The laws of concurrent list are only enforced by
(b) 1, 2 and 4 only the Supreme court.
(c) 2, 3, 4 and 6 only

https://www.meritnest.com Page 12
Polity Level-2 Question Bank Historical Background, Constituent Assembly, Salient Features

(c) Supreme Court has replaced the British Privy


4. Lucknow pact, 1916
Council as the highest court: of appeal.
(d) The decisions made by higher courts are binding Select the correct answer using the code given
on the lower courts. below:
(a) 3 and 4 only
67. Consider the following statements:
(b) 1, 2 and 4 only
1. The original Constitution was handwritten in (c) 2, 3 and 4 only
Hindi and English. (d) 1 and 2 only

2. The 58th Constitutional Amendment Act


70. “This act made the Governor-General of
provided for the translation of the Constitution in
Bengal as the Governor-General of India. It
Hindi language to be published under the authority
deprived the Governors of Bombay and Madras of
of President.
their legislative powers. It attempted to introduce
a system of open competition for selection of civil
Which of the statements given above is/are
servants.”
correct?
(a) 1 only Which one of the following acts is being described
(b) 2 only in the above given paragraph?
(c) Both 1 and 2 (a) Charter Act of 1833
(d) Neither 1 nor 2 (b) Indian Councils Act of 1861
(c) Government of India Act of 1858
68. Consider the following statements with (d) Government of India Act of 1919
reference to the concept of ‘Separation of
Powers’:
71. Which of the following statements proves that
the authority of Indian Constitution is higher than
1. It is an organisational structure where
that of Parliament?
responsibilities and powers are divided between
groups rather than being centrally held. (a) The Constitution makers were more eminent
katya124905@gmail.com
leaders than the Members of Parliament.
2. The doctrine of Separation of Powers is the basis (b) The Constitution of India was framed before the
of the American Presidential system. Parliament came into existence.
(c) Indian Constitution provides for powers and
3. Separation of Powers between the legislature,
formation of the Parliament.
executive and judiciary is an element of basic
structure of Indian Constitution.
oslo (d) The Constitution gives the power to Parliament
to alter its Basic Structure.
How many of the above given statements are
9654469135
correct? 72. The Government of India Act of 1935 had
provision of establishing an All-India Federation.
(a) Only one
In this context, how this act differentiated Indian
(b) Only two Princely States from the Governor's Provinces?
(c) All three
(d) None 1. Indian Princely states were mandatorily needed
to join Indian federation whereas it was voluntary in
case of Governor's Provinces.
69. Which of the following advocated for universal
adult franchise during pre-independence period in
2. Unlike Princely States, in case of Governor's —
India?
Provinces the federal authority was extended to
only legislative matters and not to executive
1. The Constitution of India Bill, 1895
matters.
2. Nehru report, 1928
Which of the statements given above is/are
correct?
3. Government of India act, 1919

https://www.meritnest.com Page 13
Polity Level-2 Question Bank Historical Background, Constituent Assembly, Salient Features

(a) 1 only 3. Union Constitution Committee — Dr. Rajendra


(b) 2 only Prasad
(c) Both 1 and 2
4. Minorities sub-committee — J.B. Kriplani
(d) Neither 1 nor 2 oslo
katya124905@gmail.com
How many pairs given above are correctly
73. In July 1946, the Indian National Congress matched?
appointed an ‘Experts Committee. With reference
(a) Only one
to this Experts Committee of Congress, consider
the following statements: (b) Only two
(c) Only three
1. It was appointed to prepare material for the (d) All four
Constituent Assembly.
76. Which of the following features of constitution
2. It recommended the draft of the Objectives
of India has been borrowed from Government of
Resolution.
India Act,1935?
3. Krishna Kriplani was the chairman of this (a) Fundamental rights, independence of judiciary,
committee. judicial review, impeachment of the president.
(b) Federal Scheme, Office of governor, Judiciary,
How many statements given above are correct? Public Service Commissions, Emergency provisions.
(a) Only one (c) Concurrent List, freedom of trade, commerce
(b) Only two and inter-course, and joint sitting of the two Houses
of Parliament.
(c) All three
(d) Parliamentary government, Rule of Law,
(d) None
legislative procedure, single citizenship, cabinet
system, prerogative writs.
74. With reference to the ‘Objectives Resolution,
1946’, consider the following statements:
77. Consider the following statements regarding
Federalism In India:
1. It served as the inspiration for Preamble in the
constitution.
1. The new economic policy of 1991 helped in
strengthening federalism in India.
2. It envisioned Popular Sovereignty for the
independent Indian nation.
2. Judicial Supremacy forms an essential part of
9654469135
federal polity.
3. It provided for vesting of residuary powers with
the provinces.
3. Indian model of federalism is “Coming together’,
unlike US model which is “Holding together”.
How many of the above given statements are
correct?
How many of the above given statements are
(a) Only one correct?
(b) Only two (a) Only one
(c) All three (b) Only two
(d) None (c) All three
(d) None
75. Consider the following pairs regarding the
various committees of the Constituent Assembly
78. Why India is sometimes referred to as a
and their Chairpersons:
‘quasi-federal’ polity?
1. Union Powers Committe — Jawaharlal Nehru (a) In India, executive is responsible to the
legislature for all its acts and policies.
2. Provincial Constitution Committee — Dr. B.R. (b) Indian constitution provides a Bicameral
Ambedkar legislature for the Centre and the states.

https://www.meritnest.com Page 14
Polity Level-2 Question Bank Historical Background, Constituent Assembly, Salient Features

(c) India is a federal country with balance of power (d) The head of the government cannot be changed
tilted towards Centre. without election.
(d) Indian constitution provides for division of
power between the Centre and the states. 82. ‘The people of India hold the power to freely
conduct the affairs of the nation-state. India is an
79. With reference to the difference between East independent state and there is no authority above
India Company Rule and British Crown Rule in it’.
India, consider the following statements:
Which of the following term best defines this
1. While local government did not have any attribute of India as a nation-state?
financial powers under East India Company rule, (a) Sovereignty
they were given power to impose taxes under
(b) Secularism
Crown rule.
(c) Republic
2. While East India Company rule completely (d) Democracy
excluded Indians from civil services, Crown rule
paved the way for their recruitment in 83. In the context of British rule in India, consider
administration. the following statements regarding the Amending
Act of 1781:
Which of the statements given above is/are
correct? 1. It exempted the servants of the company from
(a) 1 only the jurisdiction of the Supreme Court for their
official actions.
(b) 2 only
(c) Both 1 and 2 2. It excluded the revenue matters from the
(d) Neither 1 nor 2 jurisdiction of the Supreme Court.

80. With reference to the Regulating Act of 1773, 3. It required the courts to administer a uniform law
consider the following statements: for the people of all religions and communities.

1. It made the governors of Bengal, Bombay and How many statements given above are correct?
Madras presidencies independent of one another. (a) Only one
(b) Only two
2. It provided for the establishment of High Courts
at Calcutta, Bombay and Madras. (c) All three
(d) None
3. It recognized, for the first time, the political and
administrative functions of the company. How 84. Which of the following statement(s) correctly
many statements given above are correct? portray the secular features of Indian state as
oslo
(a) Only one mentioned in the Constitution?
(b) Only two
1. Freedom of transmission and dissemination of
(c) All three one’s religious beliefs to others is guaranteed under
(d) None Indian Constitution.
9654469135
81. The main advantage of the parliamentary form 2. Indian state cannot spend the money collected by
of governments is that: way of tax for the promotion of any particular
katya124905@gmail.com
religion.
(a) The executive and legislature work
independently. 3. Constitution prohibits imparting of religious
(b) It provides continuity of policy and is more instruction by any educational institutions in India.
efficient.
(c) The executive remains responsible to the Select the correct answer using the codes given
legislature. below:
(a) 1 only

https://www.meritnest.com Page 15
Polity Level-2 Question Bank Historical Background, Constituent Assembly, Salient Features

(b) 1 and 2 only


(c) 2 and 3 only 88. Which of the following statements regarding
(d) 1, 2 and 3 the powers of the Legislature in British India is/
are correct?
85. What is the primary objective of including the
1. The Indian Councils Act of 1861 empowered the
feature of Separation of Power in the Constitution
legislature to discuss and vote upon selected items
of India?
of the budget for the first time.
(a) To help the judiciary in providing complete
justice to the weaker sections of society. 2. The members were allowed to ask supplementary
(b) To improve the division of Labor and Functional questions for the first time by the Indian Councils
specialisation of skills. Act, 1909.
(c) To provide Checks and Balances to prevent the
3. The Government of India Act of 1919 gave the
katya124905@gmail.com
misuse of power.
legislators the right to move adjournment motions
(d) To promote fast decision making by limiting the and raise urgent issues of public interest.
encroachment of powers between the branches.
Select the correct option using the code given
86. Which among the following provisions of the below:
Constitution came into force on the date of
(a) 1 and 2 only
adoption of the Constitution?
(b) 2 and 3 only
1. Superintendence, direction and control of (c) 1 and 3 only oslo
elections to be vested in the Election Commission of (d) 1, 2 and 3
India.
9654469135
89. Which of the following functions are served by
2. Proclamation of the National Emergency.
the Constitution of India?
3. Citizenship at the commencement of the
1. It serves as guide to the state to institute laws
Constitution.
and policies to reduce poverty.
4. There shall be a Council of Ministers with the
2. It provides a set of basic rules for coordination
Prime Minister at the head to aid and advise the
amongst members of a society.
President.
3. It helps judiciary to decide the legality of the laws
Select the correct answer using the code given
framed by the legislatures.
below:
(a) 1, 3 and 4 only 4. It helps in ensuring that people with good morals
(b) 1, 2 and 4 only and values reach to power.
(c) 1 and 3 only
Select the correct answer using the code given
(d) 2, 3 and 4 only below:
(a) 1, 3 and 4 only
87. Which one of the following options is correct
with reference to Evolution of civil services in (b) 1 and 3 only
India? (c) 2, 3 and 4 only
(a) Wellesley was the first to bring into existence (d) 1, 2 and 3 only
and organize the civil services in India.
(b) Cornwallis set up the Fort William college for 90. Which of the following statements is/are
training of new recruits. correct about the Constitution of India?
(c) Aitchison committee recommended for a Public
1. It sets limits on the power of the government.
Service Commission to be immediately established.
(d) Lee commission recommended 50:50 parity 2. It provides an enabling framework for the
between the Europeans and the Indians in direct government to take steps for fulfilling the
recruitment of ICS.

https://www.meritnest.com Page 16
Polity Level-2 Question Bank Historical Background, Constituent Assembly, Salient Features

aspirations and goals of society. (a) It enables the legislature to hold more power
over the executive and the judiciary.
Select the correct answer using the code given (b) It is antithetical to the concept of division of
below: powers in a political structure.
(a) 1 only (c) Article 13 is an example of the system of checks
(b) 2 only and balances in India.
(c) Both 1 and 2 (d) Single party majority in the parliament
strengthens the system of checks and balances.
(d) Neither 1 nor 2

94. With reference to the interim government


91. Consider the following statements regarding
formed in 1946, consider the following
Indian Independence Act of 1947:
statements:
1. It deprived the Viceroy of India of legislative
1. The members of the interim Government were
powers and retained the office merely as a
members of the Viceroy’s Executive Council.
ceremonial position.
2. Jawaharlal Nehru was designated as the
2. It allowed the princely states either to join India
President of the Council.
or Pakistan but denied the right to remain
independent.
Which of the statements given above is/are
incorrect?
3. It discontinued the appointment to civil services
by the secretary of state for India. (a) 1 only
(b) 2 only
4. It abolished the office of secretary of state for
(c) Both 1 and 2
India.
(d) Neither 1 nor 2
How many statements given above are correct?
95. With reference to the comparison between the
(a) Only one
Government of India Act, 1919 and the
(b) Only two Government of India Act, 1935, consider the
(c) Only three following statements:
(d) All four
1. Both the Acts provided for the establishment of
Dyarchy, but at different levels of federal polity.
92. With reference to the Pitts India Act of 1784,
consider the following:
2. While the Act of 1919 provided for indirect
elections, the Act of 1935 provided for direct
1. It created the Board of Control to manage the
elections to the central legislature.
commercial affairs.
oslo
3. While the Act of 1919 introduced Bicameralism
2. It empowered the Court of Directors to supervise
at the Centre, the Act of 1935 introduced it in
and direct all operations of the civil and military
selected Provinces as well.
government.
How many of the above given statements are
Which of the statements given above is/are 9654469135
correct?
correct?
(a) Only one
(a) 1 only katya124905@gmail.com
(b) Only two
(b) 2 only
(c) All three
(c) Both 1 and 2
(d) None
(d) Neither 1 nor 2

93. With reference to the system of checks and


balances in the constitution, which of the
following statements is correct?

https://www.meritnest.com Page 17
Polity Level-2 Question Bank Historical Background, Constituent Assembly, Salient Features

Incorrect, Statement 2:
The legal philosophy considered opposite to
originalism is the 'living constitution' or
'modernism'. Advocated by figures like the late
Justice Ginsburg, this approach posits that the
constitution should evolve over time to address
changing societal needs.

2. Consider the following pairs of features of the


Indian Constitution and the countries from which
they were borrowed:

1. Cabinet system — US Constitution

2. Residual powers with the Centre — Canadian


Constitution

3. Supreme Court's advisory jurisdiction — French


Constitution

4. Freedom of trade, commerce and intercourse


— Japanese Constitution

How many pairs given above are correctly


1. With reference to legal philosophy of the matched?
Constitution, consider the following statements:
(a) Only one
1. ‘Modernism’ demands that the constitution (b) Only two
should be interpreted as it was understood at the (c) Only three
time when it was ratified. (d) All four

2. ‘Originalism’ demands that the constitution


Solution 2. Correct Option: (a)
should be updated with times to encompass
changing societal needs. The Constitution of India: Borrowed Features

Which of the above statement(s) is/are correct? India's Constitution incorporates elements from
the constitutions of various countries, crafting a
(a) 1 only
comprehensive and diverse legal framework.
(b) 2 only
(c) Both 1 and 2 Incorrect and Correct Matches of Borrowed
oslo

(d) Neither 1 nor 2 Features:

Pair 1 Incorrect: The Cabinet system is from the


Solution 1. Correct Option: (d)
British constitution.
Incorrect, Statement 1: 9654469135
Pair 2 Correct: Vesting of residual powers in the
Originalism in legal philosophy dictates that judges Centre comes from the Canadian constitution.
must interpret the constitution based on its katya124905@gmail.com
Pair 3 Incorrect: The Advisory jurisdiction of the
understanding at the time of ratification, regardless
Supreme Court is not derived from the Canadian
of their personal views on the case outcome.
constitution.
Originalists argue the constitution's meaning is
fixed at its inception, grounded in either the Pair 4 Incorrect: Features such as the Concurrent
language used or the framers' intentions, List, freedom of trade, commerce, and intercourse,
mandating that courts adhere to this original and the joint sitting of the two Houses of
meaning. Parliament are indeed borrowed from the
Australian Constitution.

https://www.meritnest.com Page 18
Polity Level-2 Question Bank Historical Background, Constituent Assembly, Salient Features

and passing regular laws, effectively serving as the


Additional Borrowed Features:
first Parliament of independent India.
From the British Constitution:
Incorrect Statement 3: Contrary to the claim, the
Parliamentary government, Rule of Law, legislative Act did not abolish the office of the Viceroy but
procedure, single citizenship, cabinet system, replaced it with a Governor-General, appointed by
prerogative writs, parliamentary privileges, and the British Monarch upon the dominion cabinet's
bicameralism. recommendation.

From the U.S. Constitution:


4. Consider the following pairs of features
Fundamental rights, independence of the judiciary, borrowed and the sources, with reference to the
judicial review, presidential impeachment, removal Constitution of India:
of Supreme Court and high court judges, and the
post of vice-president. 1. Concurrent List — Canadian Constitution

From the French Constitution: 2. Method of President's election — Irish


Constitution
The Republic, and the ideals of liberty, equality, and
fraternity as outlined in the Preamble.
3. Procedure established by law — Australian
Constitution
From the Canadian Constitution:
Federation with a strong Centre, vesting of 4. Procedure to amend the Constitution — South
residuary powers in the Centre, appointment of African Constitution
state governors by the Centre, and advisory
jurisdiction of the Supreme Court. How many of the pairs given above are correct?
(a) Only one
3. Consider the following statements with (b) Only two
reference to the Indian Independence Act of 1947:
(c) Only three
1. The Act made the Constituent Assembly of India (d) All four
a fully sovereign body.
Solution 4. Correct Option: (b)
2. It made the Constituent Assembly the first
Pair 1 (Incorrectly Matched):
Parliament of free India.
The feature of a concurrent list is borrowed from
3. The office of the Governor-General of India was the Australian Constitution.
abolished by the Act.
Other features from the Australian Constitution
include:
How many statements given above are correct?
Freedom of trade, commerce, and intercourse.
oslo
(a) Only one
(b) Only two Joint-sitting of the two Houses of Parliament.
(c) All three
Pair 2 (Correctly Matched):
(d) None
The method of election of the president is
9654469135
borrowed from the Irish Constitution.
Solution 3. Correct Option: (b)
Additional features from the Irish Constitution
Correct Statement 1: The Indian Independencekatya124905@gmail.com
Act
encompass:
of 1947 established the Constituent Assembly as a
sovereign entity, granting it the authority to draft Directive Principles of State Policy.
any constitution and the power to amend or repeal
Nomination of members to the Rajya Sabha.
any British law concerning India.
Pair 3 (Incorrectly Matched):
Correct Statement 2: Additionally, the Act
designated the Assembly as a legislative body The feature of the procedure established by law is
responsible for both creating India’s Constitution borrowed from the Japanese Constitution.

https://www.meritnest.com Page 19
Polity Level-2 Question Bank Historical Background, Constituent Assembly, Salient Features

Pair 4 (Correctly Matched): 6. Which of the following provisions of the Indian


Constitution shows that India is a Secular State?
The procedure for amendment of the Constitution
is borrowed from the South African Constitution.
1. The State shall endeavour to secure for all the
Another feature from the South African citizens a Uniform Civil Code.
Constitution:
2. Religious instructions shall be provided in
Election of members of the Rajya Sabha.
educational institutions maintained by the State.

5. Which of the following explains the reason 3. Religious denomination shall have the right to
behind inclusion of emergency powers in the manage its own religious affairs.
Indian constitution?
4. Citizens shall not be discriminated on the
1. To safeguard federal character of constitution grounds of religion in respect of employment under
the State.
2. To safeguard unity and integrity of country
Select the correct answer using the code given
3. To safeguard Fundamental rights of citizens below:
(a) 1, 2 and 3 only
4. To safeguard democratic political system
(b) 3 and 4 only
Select the correct answer using the code given
9654469135 (c) 1, 3 and
oslo4 only
below: (d) 1 and 4 only
(a) 1, 2, 3 and 4
(b) 2 and 4 only Solution 6. Correct Option: (c)
(c) 1, 2 and 3 only The Constitution of India embodies the principle of
(d) 2, 3 and 4 only a Secular State, meaning it does not endorse any
particular religion as the state religion.
Solution 5. Correct Option: (b)
Correct Statement 1: Article 44 aims for the State
Emergency provisions in India's Constitution, to ensure a Uniform Civil Code for all citizens,
spanning from Article 352 to 360 in Part XVIII, underscoring India’s secular nature.
empower the Central Government to tackle
exceptional situations efficiently. The core Incorrect Statement 2: Article 28 specifies that
purposes behind these provisions include: state-maintained educational institutions shall not
provide religious instruction, affirming secularism.
Preservation of sovereignty, unity, integrity, and
security of the country Correct Statement 3: Article 26 protects the rights
of religious denominations to manage their own
Protection of the democratic political system and
affairs, reflecting the secular ethos of the
Constitution
Constitution.
Inaccuracy in Statement 1: Contrary to
Correct Statement 4: Article 16 guarantees equal
safeguarding federal features, emergency
employment opportunities without discrimination
provisions centralize power with the Central
on various grounds including religion, showcasing
Government, effectively transitioning the federal
the secular framework of the Constitution.
katya124905@gmail.com
structure to a unitary one without amending the
Constitution.
7. Which of the following features ensures
Inaccuracy in Statement 3: Emergency provisions constitutionalism in India?
enable the suspension of some fundamental rights
upon presidential notification. H.V. Kamath 1. Rule of law
cautioned that this could render fundamental rights
ineffective and undermine the Constitution's 2. Separation of power
democratic foundation.

https://www.meritnest.com Page 20
Polity Level-2 Question Bank Historical Background, Constituent Assembly, Salient Features

3. Judicial review In 1961, India acquired Goa, along with Daman and
Diu, from the Portuguese through a police action.
Select the correct answer using the code given
The regions were constituted as a union territory by
below:
the 12th Constitutional Amendment Act in 1962.
(a) 1 and 2 only
Goa was granted statehood in 1987, turning Daman
(b) 2 and 3 only and Diu into a separate union territory.
(c) 1 and 3 only
(d) 1, 2 and 3 Incorporation of Sikkim into the Indian Union:
Sikkim sought closer ties with India, leading to the
Solution 7. Correct Option: (d) 35th Constitutional Amendment Act in 1974, which
made it an ‘associate state’.
Constitutionalism encompasses a governance
model centered on limited government and the rule A 1975 referendum favored Sikkim's integration
of law. This concept is manifest in India through: into India and the abolishment of the Chogyal
monarchy.
Rule of Law: The Indian Constitution enshrines the
Following the referendum, the 36th Constitutional
principles of equality, justice, liberty, and fraternity,
Amendment Act in 1975 made Sikkim the 22nd
embodying the rule of law as highlighted in the
state of India.
Preamble.
Puducherry's Transition to Indian Territory:
Separation of Power: Articles 245, 246, and
Schedule VII of the Indian Constitution distinctly France transferred the control of Puducherry
delineate legislative and executive powers between (including Karaikal, Mahe, and Yanam) to India in
the union and state governments, ensuring a clear 1954.
separation of powers.
Initially managed as an ‘acquired territory,’
Puducherry was designated a union territory by the
Judicial Review: Article 13(2) declares any law
14th Constitutional Amendment Act in 1962.
inconsistent with Part III of the Constitution null
and void. The judiciary, particularly through Articles
32 and 226, plays a crucial role in safeguarding and 9. Consider the following statements:
asserting the constitutionality of laws and the
protection of fundamental rights, with the Supreme 1. The power of Judicial review of the Supreme
and High Courts acting as the primary guardians. Court in India is narrower than that of the Supreme
Court in the United States.
8. Consider the following statements:
katya124905@gmail.com 2. The American constitution provides for
procedure established by law against that of due
1. Daman and Diu were acquired by India from the
oslo process of law provided by the Indian constitution.
Portuguese by police action.
Which of the above Statement(s) is/are correct?
2. Sikkim was added to India through referendum.
(a) 1 only
3. Puducherry was administered by India as an (b) 2 only
9654469135
‘acquired territory’ till 1962.
(c) Both 1 and 2
How many statements given above are correct? (d) Neither 1 nor 2

(a) Only one


Solution 9. Correct Option: (a)
(b) Only two
Statement 1: Correct.
(c) All three
(d) None The Supreme Court of India focuses on procedural
aspects during judicial review, unlike the American
Judiciary, which examines both procedural and
Solution 8. Correct Option: (c)
substantive aspects. However, the American
Integration of Goa, Daman and Diu into India: Constitution doesn't explicitly state the concept of
judicial review.

https://www.meritnest.com Page 21
Polity Level-2 Question Bank Historical Background, Constituent Assembly, Salient Features

(b) 2 and 3 only


Statement 2: Incorrect.
(c) 1 and 3 only
The American Constitution adopts the 'due process
(d) 1, 2 and 3
of law,' contrasting with the 'procedure established
by law' found in the Indian Constitution.
Solution 11. Correct Option: (b)

10. The constitutional position of British Correct Statement 1:


territories in India was explicitly defined in which The Ninth Schedule was introduced to the
of the following Act? Constitution through the First Amendment in 1951,
(a) The Charter Act of 1813 encapsulated by the newly created Article 31B.
This, along with Article 31A, was initiated by the
(b) The Regulating Act of 1773
government to safeguard laws related to agrarian
(c) The Pitts India Act of 1784 reform and the abolition of the Zamindari system
(d) The Charter Act of 1833 from judicial review.

Solution 10. Correct Option: (a) Incorrect Statement 2:

The Charter Act of 1813 not only renewed the The Coelho case of 2007, often referred to as the
British East India Company's charter but also Ninth Schedule case, established the judiciary's
continued its rule in India. This Act specifically authority to scrutinize laws within the Ninth
outlined the constitutional status of British Schedule, making them subject to judicial review.
territories in India, declaring British crown
sovereignty over these territories. Incorrect Statement 3:
The Parliament possesses the authority to add both
Interesting Facts: parliamentary and state legislature laws to the
Ninth Schedule.
The Regulating Act of 1773 enhanced the British
government's oversight over the company by
mandating reports on revenue, civil, and military 12. With reference to the Government of India Act
matters in India. of 1935, consider the following statements:
katya124905@gmail.com
The Pitts India Act of 1784 recognized the 1. The Act provided for establishment of India as a
company’s territories as British possessions in federation of the British provinces and princely
India. states.

The Charter Act of 1833 declared that the company 2. The Act introduced bicameralism in the British
held its territories in India 'in trust for His Majesty, provinces.
oslo
his heirs, and successors'.
3. The Act designated the Governor-General of
9654469135 India as a nominal head of the State.
11. Which of the following statements are
incorrect regarding Ninth Schedule of the How many statements given above are correct?
Constitution?
(a) Only one
1. It was added to the constitution by the first (b) Only two
Amendment. (c) All three
(d) None
2. Currently the laws placed under it are outside the
purview of judicial review.
Solution 12. Correct Option: (a)
3. Acts of the State legislatures cannot be put under Statement 1: Incorrect
it by the Parliament.
The Government of India Act of 1935 aimed to
Select the correct answer using the code given establish an All-India Federation that included
below: provinces and princely states as units. This
arrangement divided powers into three lists
(a) 1 and 2 only between the Centre and the units, with residual

https://www.meritnest.com Page 22
Polity Level-2 Question Bank Historical Background, Constituent Assembly, Salient Features

powers given to the Viceroy. However, the


federation was never realized because the princely Statement 3: Correct
states did not join.
The Act established an open competition system for
the selection and recruitment of civil servants,
Statement 2: Correct
including Indians.
The 1935 Act introduced a bicameral legislature in
This led to the appointment of the Macaulay
six of the eleven provinces, specifically Bengal,
Committee (the Committee on the Indian Civil
Bombay, Madras, Bihar, Assam, and the United
Service) in 1854.
Provinces, adding a second legislative chamber to
each.
14. Which of the following provisions of the
Statement 3: Incorrect Government of India Act of 1919, were similar to
the actual practices adopted by the Constitution
It was the Indian Independence Act of 1947, not the
of independent India with some changes? katya124905@gmail.com
Government of India Act of 1935, that designated
the Governor-General of India and the provincial
1. Direct elections to the central legislative councils.
governors as nominal (constitutional) heads, who
would act based on the advice of the council of
2. Division of powers in Federal, Provincial and the
ministers.
Concurrent List.

13. Which of the following statements is/are 3. Bicameral legislature.


correct regarding the Charter Act 1853?
4. Dyarchy at the Central government.
oslo
1. It introduced local representation in the Indian
Legislative Council for the first time. Select the correct answer using the code given
9654469135 below:
2. For the first time, it authorized the Local
(a) 1 and 3 only
Governments in India to impose taxes on persons.
(b) 2 and 4 only
3. The Act provided for an open competition system (c) 1, 2 and 3 only
of selection and recruitment of civil servants. (d) 3 and 4 only

Select the correct answer using the code given


Solution 14. Correct Option: (a)
below:
Government of India Act, 1919:
(a) 1 only
(b) 2 and 3 - Introduced bicameralism and direct elections.
(c) 1 and 2 - Replaced the Indian Legislative Council with a
(d) 1 and 3 bicameral legislature (Council of State and
Legislative Assembly).
Solution 13. Correct Option: (d) - Majority of members in both houses were directly
elected.
Statement 1: Correct
The Charter Act of 1853 introduced local Constitution of Independent India:
representation in the Indian (Central) Legislative
- Retained the practices of direct elections, upper
Council for the first time.
and lower houses.
It added six legislative members to the Governor
General's council, with four being appointed by the Misattribution to Government of India Act, 1919:
local (provincial) governments of Madras, Bombay,
- It was the Government of India Act, 1935 that
Bengal, and Agra.
divided powers between the Centre and units into
Federal, Provincial, and Concurrent Lists, assigning
Statement 2: Incorrect
residuary powers to the Viceroy.
It was the Charter Act of 1813, not the Charter Act
of 1853, that authorized Local Governments in Dyarchy at the Centre:
India to impose and enforce taxes.

https://www.meritnest.com Page 23
Polity Level-2 Question Bank Historical Background, Constituent Assembly, Salient Features

- Incorrectly attributed to the Government of India constitutional matters.


Act of 1919. It was actually introduced by the
Government of India Act, 1935, dividing federal Statement 4: Incorrect
subjects into reserved and transferred categories.
The Indian Judicial System possesses the authority
to issue Special Leave to Appeal extensively.
15. With reference to Judiciary in India and United
On the other hand, the American Judicial System
States of America, consider the following
lacks comprehensive powers to grant such appeals.
statements:

1. Unlike Indian Supreme Court, the Original 16. With reference to the role of constitution/
Jurisdiction of American Supreme Court is confined government in promoting equality among Indian
to Centre- state disputes only. citizens, which of the following statements is
incorrect?
2. Unlike the Supreme Court of India, the American
(a) The constitution allows for differentiating
Supreme Court does not have advisory jurisdiction.
between people to ensure equity.
3. Unlike American Supreme Court, the appellate (b) The constitution has given more preference to
jurisdiction of Indian Supreme Court is confined to Equality in comparison to liberty.
constitutional cases only. (c) The government can violate Article 14 of the
Constitution for implementing some of the
4. Unlike Indian Supreme Court, American Supreme Directive Principles of State Policy.
Court has a wide discretion to grant special leave to (d) Special assistance in the form of affirmative
appeal in any matter. action helps in promoting equality.

How many of the above given statements are not


Solution 16. Correct Option: (b)
correct?
Statement A: Correct. Article 14 prohibits class
(a) Only one
legislation but allows for reasonable classification
(b) Only two of persons, objects, and transactions, given that this
(c) Only three classification is not arbitrary, but rather based on
(d) All four katya124905@gmail.com clear and significant differences. This ensures
fairness by allowing for differentiation among
individuals when necessary.
Solution 15. Correct Option: (c)
Statement 1: Incorrect Statement B: Incorrect. The Constitution treats
Equality and Liberty as values of equal importance,
Original jurisdiction of the Indian Supreme Court is
ensuring both are given due consideration within
limited to certain
oslo federal cases.
Part III, without establishing a hierarchy between
Conversely, the American Supreme Court's original them.
jurisdiction extends9654469135
to federal cases, naval forces,
maritime activities, and diplomatic issues. Statement C: Correct. Article 31-C serves as an
exception to Article 14 by protecting laws aimed at
Statement 2: Correct implementing the Directive Principles, specifically
clauses (b) and (c) of Article 39, from being
The Indian Judicial System includes Advisory
challenged for violating Article 14. The Supreme
Jurisdiction.
Court summed this up by stating that "where
This provision does not exist within the American Article 31-C comes in, Article 14 goes out."
Judicial System.
Statement D: Correct. Affirmative action
Statement 3: Incorrect recognizes that formal legal equality is insufficient
to address deeply ingrained inequalities. To combat
The appellate jurisdiction of the Indian Supreme
and eventually eradicate such entrenched social
Court encompasses constitutional, criminal, and
disparities, affirmative action employs positive
civil cases.
measures, thereby advancing the cause of equality
In contrast, the American Supreme Court's by addressing the cumulative impact of historical
appellate jurisdiction primarily focuses on injustices.

https://www.meritnest.com Page 24
Polity Level-2 Question Bank Historical Background, Constituent Assembly, Salient Features

(d) A separate electorate awarded to members of


17. Which of the following functions were different communities after the Second Round
performed by the Constituent Assembly of India? Table Conference.

1. Ratified India’s Commonwealth membership. Solution 18. Correct Option: (d)

2. Elected first President of India. The Communal Award, announced by British Prime
Minister Ramsay MacDonald on August 16, 1932,
3. Appointed India’s first Prime Minister till the aimed to establish separate electorates in British
formation of new parliament in 1952. India. It was informed by the findings of the Indian
Franchise Committee (the Lothian Committee) and
Select the correct answer using the code given targeted various groups including:
below:
Forward Caste
(a) 1 only
Lower Caste
(b) 1 and 2 only
(c) 2 and 3 only Muslims
(d) 1, 2 and 3 Buddhists
Sikhs
Solution 17. Correct Option: (b)
Indian Christians
Jawaharlal Nehru, a member of the interim
government since 1946, was not appointed by the Anglo-Indians
oslo
9654469135
Constituent Assembly but by Lord Mountbatten, Europeans
the first Governor-General of the Dominion of
India, on August 15, 1947, as the first Prime Untouchables (now referred to as Dalits)
Minister.
19. Which of the following is/are salient features
Did you know? of the Constitution of India?

The Constituent Assembly, responsible for creating 1. Three-tier Governance model


the Constitution and enacting laws, also:
2. Judicial Supremacy over parliamentary
Ratified India's membership in the Commonwealth sovereignty
in May 1949.
Adopted the national flag on July 22, 1947. 3. Blend of Rigidity and Flexibility

Adopted the national anthem and song on January 4. Larger dependence on Conventions
24, 1950.
Elected Dr. Rajendra Prasad as the first President of 5. Single Citizenship
India on January 24, 1950.
How many statements given above are correct?

18. With reference to the history — of (a) Only two


constitutional legislation in Modern India, what (b) Only three
was meant by the term Communal Award? (c) Only four
(a) Offer of the British to create different states for (d) All five
all different religious communities at the time of katya124905@gmail.com
independence. Solution 19. Correct Option: (b)
(b) Award announced by the British to recognise
The Indian Constitution stands out for its unique
efforts of individuals to reduce communal tensions
features and spirit, distinguishing it from other
during non- cooperation movement.
countries' constitutions.
(c) Privileges in the form of large share in
government services awarded by the British to Salient Features:
communities who were loyal to them.

https://www.meritnest.com Page 25
Polity Level-2 Question Bank Historical Background, Constituent Assembly, Salient Features

Three-Tier Government: Initially, the Indian


Constitution established a dual polity system, Solution 20. Correct Option: (a)
focusing on the organization and powers of the
Features Borrowed from the United States
Centre and the states. However, the 73rd and 74th
Constitution:
Constitutional Amendment Acts of 1992 oslo
katya124905@gmail.com
introduced a third tier of government, the local
Fundamental Rights: Inspired by the Bill of Rights in
level, a feature not seen in any other constitution
the U.S. Constitution, India guarantees its citizens
worldwide.
essential rights.
Blend of Rigidity and Flexibility: The Constitution of Independence of Judiciary: Ensuring the judiciary
India strikes a balance between being rigid and operates without external pressures or influence,
flexible. It allows some provisions to be amended similar to the U.S.
through a simple majority in Parliament, akin to
Judicial Review: Allows Indian courts to declare
ordinary legislative processes, whereas others
laws unconstitutional, mirroring the U.S. system.
require a special majority in Parliament and
ratification by at least half of the states. Impeachment of the President: The process for
removing the president in India is influenced by the
Single Citizenship: Despite its federal structure, the U.S. method.
Indian Constitution provides for a single citizenship
Removal of Supreme Court and High Court Judges:
for the entire country, underscoring unity.
India adopts a procedure akin to the U.S. for
maintaining judicial accountability.
Incorrect Options (2 and 4):
Post of Vice-President: This role in India is
The notion of parliamentary sovereignty originates conceptually similar to that in the U.S., serving as
from the British system, whereas judicial the second highest executive officer.
supremacy is a principle of the American Supreme
Court. The Indian Constitution adeptly combines Did You Know?
these principles.
From the Constitution of Canada, India Borrowed:
Role of Conventions:
Federation with a Strong Centre: Emphasizing a
While constitutional conventions in Britain play a powerful central government akin to Canada's
significant role due to the uncodified nature of its federal structure.
constitution, they are less prominent in India's
Vesting of Residuary Powers in the Centre:
political framework, as the Indian Constitution is a
Residual legislative powers rest with the central
comprehensive, elaborate, and detailed document.
government, as in Canada.
9654469135

Appointment of State Governors by the Centre:


20. Which of the following features of the
India’s system of appointing governors mirrors
Constitution of India are borrowed from the
Canada's provincial governor appointment.
Constitution of the United States?
Advisory Jurisdiction of the Supreme Court: India
1. Fundamental rights. adopted the concept where the Supreme Court can
give advice on constitutional matters, similar to
2. Judicial review. Canada.

3. Ideal of justice in the Preamble. From the Constitution of USSR, India Adopted:

4. Federation with a strong Centre. Fundamental Duties: Encouraging citizens to fulfill


certain obligations, in line with the USSR’s
Select the correct answer using the code given constitution.
below:
Ideal of Justice in the Preamble: The vision of social,
(a) 1 and 2 only economic, and political justice in India’s Preamble is
(b) 3 and 4 only inspired by the USSR’s ideals.
(c) 1, 2 and 3 only
(d) 1, 2, 3 and 4

https://www.meritnest.com Page 26
Polity Level-2 Question Bank Historical Background, Constituent Assembly, Salient Features

21. Indian brand of socialism is called as 4. Soviet Constitution — Directive Principles of


‘Democratic Socialism’. State Policy.

Which one of the following options best describe How many of the above given pairs are correctly
‘democratic socialism’ of India? matched?
(a) It involves nationalisation of all means of (a) Only one
production and distribution and abolition of private (b) Only two
properties.
(c) Only three
(b) It follows the ideals of market economy and
(d) All four
pursues the path of globalisation towards economic
integration of domestic markets.
Solution 22. Correct Option: (a)
(c) It holds faith in a ‘mixed economy’ where both
public and private sectors co-exist side by side. Pair 1: Incorrect Matching
(d) It tends to lean heavily towards public sector to Borrowed Features from Irish Constitution:
create economic equality within the capitalist Directive Principles of State Policy, nomination of
setup. members to Rajya Sabha, and method of president's
election.
Solution 21. Correct Option: (c)
Pair 2: Incorrect Matching
Option C correctly identifies democratic socialism
as embracing a mixed economy where public and Borrowed Provision from Weimar Constitution of
private sectors operate together. According to the Germany: Suspension of Fundamental Rights
Supreme Court, democratic socialism aims to during Emergency.
eradicate poverty, ignorance, disease, and
inequality of opportunity. Indian socialism Pair 3: Correct Matching
combines Marxism and Gandhism, with a stronger
Borrowed Features from Australian Constitution:
inclination towards Gandhian principles.
Concurrent List, freedom of trade and interstate
commerce, and joint sitting of both Houses of
Additional Insights:
Parliament.
Communistic Socialism (State Socialism):
Pair 4: Incorrectly Attributed Features
Characterized by the nationalization of all means of
production and the elimination of private property. Correct Attribution:
Soviet Constitution: Fundamental duties and the
Capitalist Democracy: Advocates for a market
ideal of justice.
economy and embraces globalisation for the
katya124905@gmail.com
integration of domestic markets into the global Irish Constitution: Directive Principles of State
economy. oslo Policy.

Socialist Economies: Predominantly support the


23. With reference to the Communal Award,
public sector to ensure economic equality within a
consider the following statements:
capitalist framework.
9654469135
1. It was introduced on the recommendations of
22. With reference to ‘Source of Indian Simon Commission.
Constitution', consider the following pairs:
2. It introduced separate electorates for Sikhs,
1. Irish Constitution — Fundamental Duties and the Indian Christians, and Anglo-Indians for the first
ideal of justice. time.

2. Canadian Constitution — Suspending 3. It led to the signing of Poona Pact which gave
Fundamental Rights during Emergency. reserved seats to the depressed classes.

3. Australian Constitution — Joint sitting of the two How many statements given above are correct?
Houses of Parliament.
(a) Only one

https://www.meritnest.com Page 27
Polity Level-2 Question Bank Historical Background, Constituent Assembly, Salient Features

(b) Only two (c) Both 1 and 2


(c) All three (d) Neither 1 nor 2
(d) None
Solution 24. Correct Option: (c)
Solution 23. Correct Option: (a)
Statement 1: Correct. Negative liberty aims to
The Communal Award, also referred to as the establish and protect a domain where the individual
MacDonald Award, represented a plan for minority is untouchable, free to "do, be, or become" without
representation. It was announced by Ramsay external intervention. This domain represents a
MacDonald, the British Prime Minister, in August sacred minimum area free from external
1932. interference, acknowledging that human dignity
demands a space for unimpeded action.
Correction to Statement 1:
Statement 2: Correct. Positive liberty emphasizes
katya124905@gmail.com
The foundation of the Communal Award was not
that true freedom for an individual exists only
the Simon Commission's recommendation but the
within society and focuses on shaping a society that
findings of the Indian Franchise Committee, also
fosters individual development. Unlike negative
known as the Lothian Committee. While the Simon
liberty, which concerns itself solely with ensuring a
Commission dismissed the idea of a separate
non-interfered zone, positive liberty addresses the
electorate for depressed classes, it upheld seat
societal conditions necessary for such
reservation.
development.
Correction to Statement 2:
25. Indian Constitution is considered
oslo as one of the
The inception of a distinct electorate for Muslims
lengthiest in the world, which one of the following
was facilitated by the Indian Councils Act 1909, not
is not the reason for it?
the Communal Award. This was further extended to
9654469135
include Sikhs, Indian Christians, Anglo-Indians, and (a) A major part of it is based on the Government of
Europeans by the Government of India Act 1919. India Act, 1935, which was a bulky document.
The Communal Award sustained these separate (b) It accommodates provisions related to
electorates and additionally extended them to government mechanisms of both centre and states.
include depressed classes (Scheduled Castes).
(c) Fundamental principles of governance and
matters of administration are given in great detail.
Verification of Statement 3:
(d) Procedures related to preparation of electoral
Mahatma Gandhi, deeply troubled by the extension rolls and registration of political parties are
of communal representation to depressed classes, included in the Constitution.
fasted unto death in Yerawada Jail, Pune. This act of
protest resulted in the Poona Pact between
Solution 25. Correct Option: (d)
Congress leaders and the depressed classes,
preserving the Hindu joint electorate while Constitution of India's Length and Detail:
securing reserved seats for depressed classes.
The Constitution of India is notably one of the most
extensive written constitutions globally,
24. Consider the following statements with regard distinguished by its comprehensive and intricate
to the idea of liberty: detailing.

1. Negative Liberty is concerned with the Influence of the Government of India Act, 1935:
recognition of a minimum area in which the
Statement A is accurate: The extensive and detailed
individual can act unobstructed by others.
Government of India Act of 1935 influenced the
Constitution, incorporating familiar provisions to
2. Positive Liberty recognises that an individual can
ease the transition.
be free only in society and not outside of it.
Comprehensive Federal Structure:
Which of the statements given above is/are
correct? Statement B is accurate: Unlike many federal
nations where the federal government is delineated
(a) 1 only
and states draft their own constitutions, India's
(b) 2 only

https://www.meritnest.com Page 28
Polity Level-2 Question Bank Historical Background, Constituent Assembly, Salient Features

Constitution uniquely outlines both the Union and (d) 1, 2, 3 and 4


State governments with precision.
Solution 27. Correct Option: (b)
Distribution of Powers and Administrative Details:
The Second and Third Schedules of the Constitution
Statement C is accurate: The Constitution commonly include three offices:
meticulously outlines the distribution of powers
between the Union and States, including provisions Judges of the Supreme Court
for inter-State relations and administrative details,
emphasizing its role in governance beyond Judges of the High Courts
fundamental principles. Comptroller and Auditor-General of India

Representation of the People Acts: Notably, the office of the Prime Minister is not
Statement D is incorrect: The Representation of the mentioned in either the Second or Third Schedule
People Act, 1951, and its 1950 counterpart, of the Constitution.
focusing on political party registration and electoral
rolls respectively, are statutory laws enacted by 28. With reference to the Government of India Act
Parliament and do not form part of the of 1935, consider the following statements:
Constitution.
1. It provided for the adoption of the dyarchy at the
26. The “Instrument of Instructions” contained in Centre.
the Government of India Act 1935 have been
incorporated in the Constitution of India in the 2. It abolished the Council of India, established by
year 1950 as: the Government of India Act of 1858.

(a) Fundamental Rights 3. It provided for the separate electorates for


(b) Directive Principles of State Policy depressed classes, women and labour.
(c) Extent of executive power of State
How many statements given above are correct?
(d) Conduct of business of the Government of India
(a) Only one
katya124905@gmail.com
Solution 26. Correct Option: (b) (b) Only two
The Directive Principles of State Policy in the Indian (c) All three
Constitution are akin to the Instrument of (d) None
Instructions from the Government of India Act,
1935. Solution 28. Correct Option: (c)
oslo
Statement 1:
27. Which of the following offices is/are included
in the both second and third schedule of the Correct: The Act introduced dyarchy at the Centre,
9654469135
Constitution? but this system was never implemented.

1. The Judges of the High Courts Statement 2:


Correct: The Act terminated the Council of India,
2. The Judges of the Supreme Court set up by the Government of India Act of 1858,
which was designed to aid the Secretary of State for
3. The Comptroller and Auditor-General of India India with a 15-member council. Following its
dissolution, the Secretary was supported by a team
4. The Prime Minister's Office of advisors.

Select the correct answer using the code given Statement 3:


below:
Correct: The Act expanded the separate electorate
(a) 1 and 2 only system to include depressed classes, women, and
(b) 1, 2 and 3 only labor.
(c) 2, 3 and 4 only

https://www.meritnest.com Page 29
Polity Level-2 Question Bank Historical Background, Constituent Assembly, Salient Features

29. Which of the following factors make the Indian


Granville Austin commented that the Assembly was
Constitution more acceptable to the people at
essentially a Congress-dominated body, reflecting
large?
the political landscape of that time, with Congress
members occupying 82% of the seats after
1. National leaders proposed the subjection of
Partition.
constitution to referendum of people.

2. It was supported by popular leaders who enjoyed 30. Match the following pairs:
immense credibility.
Feature borrowed:
3. Constitution was framed by representatives from
1. Office of the Governor
all sections of Indian society.
2. Supreme Court's advisory jurisdiction
Select the correct answer using the code given
3. Joint sitting of the two Houses of Parliament
below:
4. Election to the Rajya Sabha
(a) 1 and 2 only
(b) 2 and 3 only Source
(c) 1 and 3 only
A. Canadian Constitution
(d) 1, 2 and 3
B. South African Constitution
Solution 29. Correct Option: (b) C. Australian Constitution
Statement 1: Incorrect D. Government of India Act, 1935

The Indian Constitution was not subjected to a Choose the correct answer using the code below:
referendum.
(a) 1-A, 2-C, 3-B, 4-D
It was backed by consensus among popular leaders, (b) 1-D, 2-A, 3-C, 4-B
contributing to its effectiveness.
(c) 1-B, 2-D, 3-A, 4-C
The citizens embraced it by adhering to its rules, (d) 1-C, 2-B, 3-D, 4-A
katya124905@gmail.com
demonstrating the constitution's authority and
effectiveness through public enactment.
Solution 30. Correct Option: (b)
Statement 2: Correct Pair 1-D: Correct. The concept of the Governor's
office was adopted from the Government of India
The framers of the Indian Constitution were Act of 1935.
individualsoslo
with immense public credibility.
Pair 2-A: Correct. The Supreme Court's advisory
They had the ability to negotiate and command
jurisdiction is inspired by the Canadian
respect across diverse societal sections, lending
9654469135
Constitution.
legitimacy and effectiveness to the constitution.
Pair 3-C: Correct. The provision for a joint sitting of
Statement 3: Correct
the two houses of Parliament is taken from the
Australian Constitution.
Despite the Constituent Assembly not being
directly elected through adult franchise, it was
Pair 4-B: Correct. The method for electing members
representative of Indian society's diverse sections,
of the Rajya Sabha is derived from the South African
including various religious groups, Scheduled
Constitution.
Castes (SCs), Scheduled Tribes (STs), and women.
Key Indian personalities were included, although
31. Consider the following statements regarding
Mahatma Gandhi was not a member.
the Second Schedule and the Third Schedule of
Indian Constitution:
Critique of the Constituent Assembly
1. Both the second and third schedules contains
Critics argue that the Constituent Assembly was
provision related to Vice President.
dominated by the Congress party.

https://www.meritnest.com Page 30
Polity Level-2 Question Bank Historical Background, Constituent Assembly, Salient Features

High Court Judges), and the CAG.


2. The President and the Governor of states are
included only in Second Schedule and not in the
The Third Schedule details the oath and affirmation
Third Schedule.
formats for Union Ministers, Election Candidates
(Parliament and State Legislature), Members of
3. The Comptroller and Auditor General of India is
Parliament (MPs), Judges (Supreme Court and High
included in both the Schedules.
Courts), Comptroller and Auditor General, and
State Ministers.
4. Both the Schedules can be amended by a simple
majority of Parliament.
32. With reference to ‘Constituent Assembly;
How many of the above given statements are consider the following statements:
correct?
1. The assembly was empowered to alter any law
(a) Only one
made by the British Parliament in relationkatya124905@gmail.com
to India.
(b) Only two
(c) Only three 2. The members of the Muslim League from the
(d) All four Indian Dominion never entered the Assembly.

3. Both H.C. Mukherjee and V.T. Krishnamachari


Solution 31. Correct Option: (c)
were elected as the Vice- Presidents of the
The Indian Constitution, adopted in 1949, initially Assembly.
included 8 Schedules but now comprises 12
Schedules. 4. Dr Sachchidanand Sinhaoslo
acted as the chairperson
of the assembly whenever it met as the legislative
Clarifications on Statements: body.
9654469135
Incorrect Statement: The Vice President is not How many of the above given statements are
listed in the Third Schedule, which includes forms of correct?
oaths and affirmations for various officials. Instead,
(a) Only one
the Vice President, serving as the Chairman of the
Rajya Sabha, is mentioned in the Second Schedule, (b) Only two
which addresses emoluments, allowances, (c) Only three
privileges, etc. (d) All four

Correct Statement: Only the Second Schedule lists


Solution 32. Correct Option: (b)
the President and the Governors of states,
excluding them from the Third Schedule. Correct Statement 1: The Indian Independence Act
of 1947 granted the Assembly full sovereignty,
Correct Statement: The Comptroller and Auditor allowing it to amend or repeal any British laws
General (CAG) of India, the leader of the Indian pertaining to India.
audit and accounts department and principal
guardian of the public purse, is included in both the Incorrect Statement 2: Following the Mountbatten
Second and Third Schedules. Plan of June 3, 1947, which led to India's partition,
representatives from princely states and the Indian
Correct Statement: Amendments to the Second and Dominion's Muslim League joined the Assembly.
Third Schedules can be made with a Simple Majority However, Muslim League members from regions
in both Houses of Parliament, independently of allocated to Pakistan exited the Assembly for India.
Article 368.
Correct Statement 3: Dr. Rajendra Prasad was
Additional Insights: appointed President of the Assembly, with H.C.
Mukherjee and V.T. Krishnamachari serving as
The Second Schedule outlines the provisions Vice-Presidents.
concerning the salaries, allowances, and privileges
of key positions including the President, Governors, Incorrect Statement 4: The Constituent Assembly
Parliamentary Speakers (Lok Sabha and Rajya functioned as free India's first Parliament. When
Sabha), Judiciary Members (Supreme Court and acting as the constituent body, Dr. Rajendra Prasad

https://www.meritnest.com Page 31
Polity Level-2 Question Bank Historical Background, Constituent Assembly, Salient Features

presided, whereas G.V. Mavlankar led its sessions 4. Twelfth Schedule — Powers, authority and
as the legislative body. Initially, Dr. Sachchidanand responsibilities of Panchayats.
Sinha was chosen as the temporary President,
emulating the French model. How many pairs given above are incorrectly
matched?
33. The Constituent Assembly appointed a number (a) Only one
of committees to deal with different tasks of (b) Only two
constitution-making. In this context, consider the
(c) Only three
following committees:
(d) All four
1. Rules of Procedure Committee
Solution 34. Correct Option: (d)
2. Steering Committee
The Indian Constitution comprises 12 Schedules,
with some pairs being incorrectly matched as
3. Finance and Staff Committee
follows:
4. Ad-hoc Committee on the National Flag
Incorrect Pair 1:
How many of the above given committees were Third Schedule: Outlines the forms of oath and
headed by Dr. Rajendra Prasad? affirmation for various officials including Union
Ministers of India, Parliament Election Candidates,
(a) Only one
Members of Parliament, Supreme Court Judges,
(b) Only two Comptroller and Auditor General, State Ministers,
(c) Only three State Legislature Elections’ Candidates, State
(d) All four Legislature Members, and High Court Judges.
Fourth Schedule: Allocates Rajya Sabha seats to
Solution 33. Correct Option: (d) states and union territories.
The Constituent Assembly established several
Incorrect Pair 2:
committees for various aspects of
constitution-making, categorizing them into eight Fifth Schedule: Pertains to the administration and
major and multiple minor committees. Among control of Scheduled Areas and Scheduled Tribes in
these, four significant committees led by Dr. states other than Assam, Meghalaya, Tripura, and
Rajendra Prasad included: Mizoram.
Sixth Schedule: Governs the administration of tribal
Rules of Procedure Committee
katya124905@gmail.com areas in Assam, Meghalaya, Tripura, and Mizoram.
Steering Committee
oslo Incorrect Pair 3:
Finance and Staff Committee
Eighth Schedule: Recognizes languages under the
Ad-hoc Committee on the National Flag
Constitution.
Seventh Schedule: Divides powers between the
34. Consider the following pairs
Union and the States into three lists: Union List (I),
9654469135
State List (II), and Concurrent List (III).
Schedules of the Constitution — Subject matter
Incorrect Pair 4:
1. Third Schedule — Allocation of seats in the Rajya
Sabha to the states. Twelfth Schedule: Describes powers, authority, and
responsibilities of Municipalities.
2. Fifth Schedule — Provisions relating to the
Eleventh Schedule: Details powers, authority, and
administration of tribal areas like in Meghalaya.
responsibilities of Panchayats.
3. Eighth Schedule — Division of powers between
the Union and the States. 35. Consider the following statements:

https://www.meritnest.com Page 32
Polity Level-2 Question Bank Historical Background, Constituent Assembly, Salient Features

1. The Act dropped the title of Emperor of India Governance Changes: It replaced the office of
from the royal titles of the King of England. Viceroy with a Governor-General for each
dominion, appointed by the British King upon the
2. The Act ended the British rule in India and dominion cabinet's advice.
declared India as an independent and sovereign oslo
katya124905@gmail.com
state. Legislative Powers: The Constituent Assemblies of
both dominions were granted the authority to draft
3. The Act provided for the appointment of some and enact their constitutions, repeal any act of the
civil servants by the Secretary of State for India. British Parliament—including the Independence Act
itself—and legislate for their territories until the
How many of the above given statements are new constitutions were implemented.
correct about the Indian Independence Act of
1947?
36. Consider the following statements:
(a) Only one
(b) Only two 1. The Constituent Assembly of India was formed
under the scheme formulated by the Cripps
(c) All three
Mission.
(d) None
2. The Constituent assembly generally followed the
Solution 35. Correct Option: (b) policy of consensus and accommodation with
respect to drafting of the Constitution.
On June 3, 1947, Lord Mountbatten, the Viceroy of
India, unveiled the Mountbatten Plan, which was
Which of the statements given above is/are
subsequently endorsed by both the Congress and
correct?
the Muslim League. The plan led to the enactment
of the Indian Independence Act, immediately (a) 1 only
putting it into effect. (b) 2 only
(c) Both 1 and 2
Correct Statements:
(d) Neither 1 nor 2
Abolition of Royal Titles and Paramountcy: The act
removed the ‘Emperor of India’ title from the British Solution 36. Correct Option: (b)
king’s royal titles and ended British paramountcy
Statement 1: Incorrect. The Constituent Assembly
over Indian princely states and treaty relations with
was formed in November 1946, following the
tribal areas from August 15, 1947.
Cabinet Mission Plan (1946).
9654469135
End of British Rule: It officially concluded British
Statement 2: Correct. The drafting process of the
sovereignty in India, establishing India as an
Constitution was characterized by two uniquely
independent and sovereign entity.
Indian political features: consensus and
accommodation. Members adhered to a
Incorrect Statement:
consensus-based decision-making process and
generally practiced a policy of accommodation.
Civil Service and Secretary of State's Office:
Contrary to the claim, the act did not abolish the
office of the Secretary of State for India nor did it 37. Consider the following pairs of Schedules of
end the appointment to civil services and the Constitution of India and its subject matter:
reservation of posts by the Secretary of State for
India. 1. Second Schedule – provisions for emoluments,
allowances, privileges
Additional Insights:
2. Fourth Schedule — forms of Oaths or
Partition and Dominion Status: The act facilitated Affirmations
the partition of India, creating two independent
dominions, India and Pakistan, each with the option 3. Seventh Schedule — Division of powers between
to leave the British Commonwealth. the Union and the States

https://www.meritnest.com Page 33
Polity Level-2 Question Bank Historical Background, Constituent Assembly, Salient Features

4. Eleventh Schedule — Powers, authority and


responsibilities of Municipalities 3. It ended the system of double government by
abolishing the Board of Control and Court of
How many pairs given above are correctly Directors.
matched? oslo
katya124905@gmail.com
4. It empowered the Viceroy to issue ordinances
(a) Only one
without the concurrence of the legislative council.
(b) Only two
(c) Only three How many of the above given statements are
(d) All four correct?
(a) Only one
Solution 37. Correct Option: (b) (b) Only two
Pair 1: Correctly Matched (c) Only three
The Second Schedule outlines the emoluments, (d) All four
allowances, and privileges for the President, State
Governors, the Lok Sabha's Speaker and Deputy Solution 38. Correct Option: (b)
Speaker, the Rajya Sabha's Chairman and Deputy
After the 1857 revolt, the British Government
Chairman, the Legislative Assemblies' Speaker and
recognized the need to involve Indians in their
Deputy Speaker, the Legislative Councils' Chairman
country’s administration. As part of this strategy,
and Deputy Chairman, Judges of the Supreme
the British Parliament passed three significant acts
Court and High Courts, and the Comptroller and
in 1861, 1892, and 1909.
Auditor General.
Correct Statements:
Pair 2: Incorrectly Matched
The Fourth Schedule details the allocation of seats Indian Councils Act of 1861: This act reintroduced
in the Rajya Sabha to the states and union legislative power to the Bombay and Madras
territories, while the Third Schedule provides the Presidencies, reversing the Charter Act of 1833's
Forms of Oaths or Affirmations. limitations.

Pair 3: Correctly Matched Further Provisions of the Indian Council Act of


1861: It allowed the Viceroy to:
The Seventh Schedule explains the division of
powers between the Union and States, including Make procedural rules for council operations.
List I (Union List), List II (State List), and List III
Implement the ‘portfolio’ system (initiated by Lord
(Concurrent List). 9654469135
Canning in 1859) for streamlined governance.
Pair 4: Incorrectly Matched Issue ordinances in emergencies without needing
legislative council approval.
The Eleventh Schedule, enacted via the 73rd
Amendment Act of 1992, specifies the powers,
Incorrect Statements:
authorities, and responsibilities of Panchayats,
covering 29 matters.
The establishment of a 15-member Council of India
The Twelfth Schedule, introduced by the 74th to aid the Secretary of State was actually done
Amendment Act of 1992, defines the powers, under the Government of India Act of 1858, not the
authorities, and responsibilities of Municipalities, Indian Councils Act of 1861. The council served an
covering 18 matters. advisory role.

The Government of India Act of 1858 eliminated


38. With reference to ‘Indian Council Act of 1861’,
the dual government system by dissolving the
consider the following statements:
Board of Control and the Court of Directors, not
the Indian Councils Act of 1861.
1. It restored the legislative powers to the Bombay
and Madras Presidencies.
39. With reference to the constitution, consider
2. It established a 15-member Council of India to the following statements:
assist the secretary of state for India.

https://www.meritnest.com Page 34
Polity Level-2 Question Bank Historical Background, Constituent Assembly, Salient Features

1. Constitutions are required only in democratic (a) B R Ambedkar was the Chief Draftsman of
countries. Constitution in the Assembly.
(b) None of the members of the Assembly were
2. Constitution is a legal document that does not Women.
deal with ideals and values.
(c) The Assembly adopted the Elephant as its
symbol/seal.
3. A constitution gives its citizens a moral identity.
(d) S Varadachari was the legal advisor to the
How many statements given above are correct? Assembly.

(a) Only one


Solution 40. Correct Option: (c)
(b) Only two
Option a is incorrect: The chief draftsman of the
(c) All three
Constitution in the Constituent Assembly was S N
(d) None Mukherjee, not Dr. Ambedkar. Dr. B R Ambedkar
served as the Chairman of the Drafting Committee.
Solution 39. Correct Option: (a)
Option b is incorrect: The Constituent Assembly
Statement 1: The claim that constitutions exist only
included 15 female members.
in democratic nations is incorrect. Constitutions are
also present in various other governmental forms,
Option c is correct: The Elephant was selected as
such as monarchies and single-party states. For
the symbol for the Constituent Assembly's seal.
instance, in a monarchical system, the monarch
often has significant decision-making power.
9654469135 oslo
Option d is incorrect:
Similarly, in some systems like the former Soviet
Union, a single party may hold the decision-making The constitutional/legal advisor appointed to the
authority. In contrast, democratic constitutions are Constituent Assembly was Sir B N Rau, not S
characterized by decision-making power vested in Varadachari.
the people.
S Varadachari led the Assembly's Ad Hoc
Committee on the Supreme Court and was not a
Statement 2: The assertion that constitutions solely
member of the Assembly.
address the form of government is inaccurate.
Beyond outlining governmental structure,
constitutions embody the ideals and values a 41. The demand for Constituent Assembly was
country strives to uphold. These ideals and values accepted for the first time by the British
are frequently reflected in the preamble, oaths, and Government through:
opening declarations. Effective constitutions
(a) Wavell plan
maintain a balance between preserving core values
and allowing for adaptations to changing (b) Simon Commission
circumstances. (c) Gandhi Irwin Pact
(d) August Offer
Statement 3: The statement that a constitution
reflects a people's fundamental identity is accurate.
Solution 41. Correct Option: (d)
A constitution establishes a collective identity
through a shared commitment to basic governance In 1935, the Indian National Congress (INC)
norms and principles. While individuals may officially demanded the establishment of a
possess various identities before a constitution, it is Constituent Assembly for the first time to draft
the agreement on these principles that forms their India's Constitution. This demand was eventually
core political identity. Additionally, constitutional acknowledged in principle by the British
katya124905@gmail.com
norms create a framework for pursuing personal Government with the 1940 August Offer.
aspirations, goals, and freedoms, thus contributing
to a shared moral identity.
42. Consider the following pairs (Member —
Portfolio) with reference to the First Cabinet of
40. Which of the following statements with Independent India (1947):
reference to the Constituent Assembly of newly
independent India is correct? 1. Jawaharlal Nehru — Home Affairs

https://www.meritnest.com Page 35
Polity Level-2 Question Bank Historical Background, Constituent Assembly, Salient Features

2. Sardar Patel — Information and Broadcasting


1. Charter Act of 1813 ended all the commercial
activities of the East India Company.
3. Rajkumari Amrit Kaur — Health
2. Charter Act of 1833 created Government of India
4. Dr. Rajendra Prasad — Education oslo
having authority over the katya124905@gmail.com
entire territorial area
possessed by the British in India.
How many of the above given pairs is/are correctly
matched?
3. Charter Act of 1833 separated the legislative and
(a) Only one executive functions of the Governor-General’s
(b) Only two council.
(c) Only three
How many statements given above are correct?
(d) All four
(a) Only one
Solution 42. Correct Option: (b) (b) Only two
(c) All three
Pair 1: Incorrect.
(d) None
Clarification: Sardar Vallabhbhai Patel was
responsible for Home Affairs, not Jawaharlal
Solution 43. Correct Option: (a)
Nehru. Nehru served as the Prime Minister,
overseeing External Affairs and Commonwealth Statement 1 Misconception Corrected: The Charter
Relations, as well as Scientific Research. Act of 1813 eliminated the East India Company's
monopoly over Indian trade, opening it to all British
Pair 2: Correct. merchants, while retaining its monopoly on tea and
China trade. The Charter Act of 1833 then
Sardar Vallabhbhai Patel managed Home,
transformed the Company into a purely
Information and Broadcasting, and States.
administrative entity, ceasing its commercial
operations.
Pair 3: Correct.
Rajkumari Amrit Kaur was assigned the Health Statement 2 Affirmed: The Charter Act of 1833
portfolio. established the Government of India, extending
authority over all British territories in India. It
Pair 4: Incorrect. upgraded the Governor-General of Bengal to the
Governor-General of India, consolidating civil and
Maulana Abul Kalam Azad was in charge of
military powers, with Lord William Bentinck serving
Education, whereas Dr. Rajendra Prasad handled
as the inaugural Governor-General.
9654469135
the Food and Agriculture portfolio.
Statement 3 Clarification: The Charter Act of 1853
Additional Members and Their Portfolios:
marked the first distinction between the legislative
and executive branches of the Governor-General’s
Dr. John Matthai: Railways and Transport.
council by introducing the Indian (Central)
Sardar Baldev Singh: Defence. Legislative Council, a distinct legislative entity.
Jagjivan Ram: Labour.
44. With reference to the Indian and British
C.H. Bhabha: Commerce.
Parliamentary System, consider the following
Rafi Ahmad Kidwai: Communications. statements:
B.R. Ambedkar: Law.
1. Both the Indian and British Parliamentary
Shri R.K. Shanmukham Chetty: Finance. systems are based on Doctrine of the Sovereignty
of Parliament.
Dr. Syama Prasad Mukherji: Industries and
Supplies.
2. In both the systems, the Prime Minister must be
Shri N. V. Gadgil: Works, Mines and Power. from the lower house of Parliament.

3. Britain has the system of legal responsibility of


43. Consider the following statements:
the ministers, while India has no such system.

https://www.meritnest.com Page 36
Polity Level-2 Question Bank Historical Background, Constituent Assembly, Salient Features

(b) 1 and 4 only


4. Both the systems have an elected Head of the
(c) 1, 2 and 3 only
State.
(d) 2, 3 and 4 only
How many of the above given statements are
correct? Solution 45. Correct Option: (b)
(a) Only one In 1946, following the Cabinet Mission's
(b) Only two recommendations, India formed the Constituent
Assembly to draft its constitution, establishing a
(c) All three
sovereign body.
(d) None
Correct Statements:
Solution 44. Correct Option: (a)
Critique of Representation:
The Indian parliamentary system, though largely
inspired by the British model, exhibits significant Critics argue that the Constituent Assembly was
differences: not fully representative since its members were not
directly elected by the people through universal
Sovereignty: Unlike Britain's Parliament, which is adult suffrage, making it a partly elected and partly
sovereign, the Indian Parliament operates under a nominated entity.
written Constitution, enforcing limited powers due
to its federal system, judicial review, and the Domination by Congress:
protection of fundamental rights.
The Constituent Assembly was seen as being
dominated by the Congress party. Granville Austin
Prime Minister's Membership: In contrast to the
highlighted this dominance by stating the Assembly
British requirement for the Prime Minister to be a
was essentially the Congress, and in turn, the
member of the Lower House, the Indian Prime
Congress represented India.
Minister can be a member of either House of
Parliament.
Incorrect Statements:
Ministerial Responsibility: India does not mandate
Legislative Function:
legal responsibility for ministers to countersign
official acts of the Head of State, a practice common According to the Indian Independence Act of 1947,
in Britain, highlighting a key difference in the the Constituent Assembly also served as a
operational dynamics of the two systems. legislative body, playing a crucial role in the
enactment of laws until the first Lok Sabha was
Form of Government: India adopts a republican formed.
system with an elected President, moving away
from Britain's hereditary monarchy, underscoring a Lawyer-Politician Dominance:
fundamental structural divergence between the
Critics maintain that the Constituent Assembly was
two governance models. oslo
predominantly composed of lawyers and politicians,
which they argue led to the Constitution's
45. Which of the following are the valid criticisms complexity and length.
of constituent assembly of India?
46. Which of the following are the reasons for the
9654469135
1. Members are not directly elected by people
inclusion of emergency provisions in the Indian
constitution?
2. Nonfunctional as a legislative body katya124905@gmail.com

1. To safeguard the sovereignty, unity, integrity and


3. Lack of representation to legal professionals
security of the country.
4. Dominated by single political party
2. To vest wide powers to centre so as to handle
special situations.
Select the correct answer using the code given
below:
3. To prevent maladministration or corruption in
(a) 1 only the country.

https://www.meritnest.com Page 37
Polity Level-2 Question Bank Historical Background, Constituent Assembly, Salient Features

Constitutions are categorized into two types: rigid


4. To sort out intra-party problems of the ruling and flexible. A rigid constitution requires a
party. specialized procedure for amendments, exemplified
by the American Constitution. Conversely, a
Select the correct answer using the code given flexible constitution can be amended as easily as
oslo
katya124905@gmail.com
below: ordinary laws are enacted, seen in the British
Constitution.
(a) 1 and 2 only
(b) 1, 2 and 3 only The Indian Constitution is a blend of both rigid and
(c) 1, 3 and 4 only flexible characteristics.
(d) 2 and 3 only
It incorporates elements of rigidity and flexibility in
amending procedures.
Solution 46. Correct Option: (a)
Article 368 outlines two amendment methods: one,
Statement 1 is correct: The emergency provisions in
certain provisions need a special majority in
the Indian constitution were designed to protect
Parliament; two, other provisions require a special
the nation’s sovereignty, unity, integrity, and
majority in Parliament plus ratification by at least
security, preserve the democratic framework, and
half of the states.
uphold the Constitution.

Statement 2 is correct: These provisions grant the 48. With reference to the composition of India’s
central government extensive powers to manage Constituent Assembly, consider the following
exceptional circumstances, allowing it to assume statements:
complete legislative and executive authority over
states and, if necessary, limit or suspend citizens' 1. It was constituted on the basis of
freedoms during an emergency. recommendations of the Cabinet Mission.

Statement 3 is incorrect: The purpose of the 2. Members from the Princely States were drawn
emergency provisions was not to address through nomination.
maladministration or corruption but to prevent the
collapse of the constitutional system. Corruption 3. Seats were reserved for various minorities but
does not qualify as a ground for declaring an there was no separate electorate.
emergency.
How many of the above given statements are
Statement 4 is incorrect: The provisions were not correct?
meant to resolve issues within the ruling party but
(a) Only one 9654469135
to counteract internal subversion and actions
against the Constitution. (b) Only two
(c) All three
47. Which one of the following is a reason why (d) None
Indian Constitution is described as the blend of
rigidity and flexibility? Solution 48. Correct Option: (b)
(a) Both the Central and the state governments can Statement 1: True
initiate the amendment to the Constitution.
Formation: Constituted in November 1946, as
(b) Ordinary Legislative power of the Parliament is outlined by the Cabinet Mission Plan.
superior to its Constituent power.
Cabinet Mission's Decision: Rejected the Muslim
(c) Amendments to the Constitution can be made
League's demand for separate assemblies, opting
both through a special procedure and through the
instead for a single assembly with 389 members.
manner in which ordinary laws are enacted.
(d) Private members cannot introduce Statement 2: True
constitutional amendment bill.
Composition: A mix of indirectly elected and
nominated members.
Solution 47. Correct Option: (c)
Selection Process: Members from British Indian
provinces were elected, whereas members from

https://www.meritnest.com Page 38
Polity Level-2 Question Bank Historical Background, Constituent Assembly, Salient Features

Princely States were nominated. communities, allowing individuals to practice the


religion of their choice and enabling religious
Statement 3: False minorities to preserve their culture and educational
institutions.
Minority Protection Method: The British
introduced Separate Electorates for indirect
Correct Statement: Indian secularism supports
Constituent Assembly elections, allowing Sikh and
state-endorsed religious reforms, evidenced by the
Muslim community members to be elected by their
Constitution's ban on untouchability and laws
own communities from the Provincial Legislatures.
against child marriage and inter-caste marriage
Separate reservations were allocated for these
bans rooted in Hinduism.
communities.
Correct Statement: Beyond separating church and
Did You Know? Women in the Constituent
state, Indian secularism values inter-religious
Assembly
equality. It addresses both intra-religious and
Representation: The Assembly included 15 women inter-religious domination, opposing the
representatives. mistreatment of dalits and women in Hinduism,
discrimination in Indian Islam and Christianity, and
Notable Members: Among them were Sarojini
threats from the majority community to minority
Naidu, Vijay Lakshmi Pandit, Durgabai Deshmukh,
religious rights.
and Rajkumari Amrit Kaur, all contributing to the
constitution-making process.
Western Secularism

49. With reference to Indian form of secularism, Incorrect Statement: Contrary to Indian secularism,
consider the following statements: Western secularism prohibits state assistance to
religious institutions, including financial support for
1. Indian secularism deals only with religious educational establishments affiliated with religious
freedom of individual and not of community. groups. However, the Indian Constitution permits
state aid to religious institutions but forbids using
2. Indian secularism allows the state-supported public tax revenue to support or maintain any
religious reform. specific religion.

3. Indian secularism focuses on both inter and intra


50. With reference to the Constitution and its
religious equality.
framework, consider the following statements:
4. Under Indian secularism the state cannot aid any
1. A written, or unwritten constitution needs to
religious institution.
katya124905@gmail.com
exist tor a country to be called a nation.
How many statements given above are correct?
2. A constitution specifies who has the power to
(a) Only one oslo make decisions in a society.
(b) Only two
3. A constitution provides a set of basic rules that
(c) Only three
allow for minimal coordination amongst members
(d) All four of a society.
9654469135

Solution 49. Correct Option: (b) 4. A constitution provides an identity to its citizens.
Secularism Overview
How many of the above given statements are
correct?
Secularism is the principle that religion should not
influence or intervene in the organization of (a) Only one
society, including aspects such as education and (b) Only two
government.
(c) Only three
Indian Secularism (d) All four

Incorrect Statement: Indian secularism embraces Solution 50. Correct Option: (c)
religious freedom for individuals and minority

https://www.meritnest.com Page 39
Polity Level-2 Question Bank Historical Background, Constituent Assembly, Salient Features

A constitution defines the foundational principles backgrounds.


and laws for a nation, state, or group, outlining the
government's powers and responsibilities, and Correct Statement 3: Asymmetry in governance is
assuring certain rights to its citizens. formalized through legal and institutional
frameworks. In India, this is anchored in the
Statement 1: Incorrect. A nation is characterized by constitution, allowing certain states like Nagaland
a shared sense of unity and identity among its and Mizoram special provisions under Article 37.
people, not necessarily by having a constitution. Similarly, asymmetries exist at the sub-state level
with Union Territories. Conversely, countries like
Statement 2: Correct. Constitutions establish who Indonesia and China employ legislative means to
holds decision-making power, detailing the establish asymmetry.
structure of government and the legislative
process.
52. Arrange the following events in the
chronological order of their occurrence:
Statement 3: Correct. Constitutions provide
essential rules for societal coordination, setting
1. Adoption of National Anthem
publicly known guidelines necessary for group
functionality.
2. Ratification of Commonwealth membership
Statement 4: Correct. Constitutions affirm a
3. Adoption of National Flag in its present form
collective identity, offering political and social
recognition while embodying core values and
4. Adoption of National Emblem
granting moral identity through citizenship.
Select the correct answer using the codes given
51. Consider the following statements regarding below:
the term ‘Asymmetric Federalism’:
(a) 1-2-4-3
1. In Asymmetric Federalism, a particular state can (b) 3-2-1-4
have greater executive powers than other states. (c) 1-3-2-4
(d) 3-4-1-2
2. It overlooks the diverse cultural, linguistic, and
historical differences among constituent units.
Solution 52. Correct Option: (b)
3. Indian Constitution provides for the Asymmetric The chronological order of the events is:
federalism.
Adoption of the National Flag:
How many of the above given statements are
The National Flag of India was officially adopted in
correct?
its current form during the Constituent Assembly
(a) Only one meeting on July 22, 1947.
oslo
(b) Only two
Ratification of India's Commonwealth Membership:
(c) All three
(d) None In May 1949, the Constituent Assembly ratified
India’s membership in the Commonwealth.
Solution 51. Correct Option: (b) 9654469135
Adoption of the National Anthem:
Correct Statement 1: Asymmetrical federalism
'Jana Gana Mana' was adopted as India's national
enables certain regions to enjoy special statuses,
katya124905@gmail.com
anthem by the Constituent Assembly on January
granting them unique executive, legislative, and
24, 1950.
sometimes judicial powers beyond those of other
states.
Adoption of the National Emblem:
Incorrect Statement 2: Asymmetric federalism The National Emblem of India, adapted from the
acknowledges the cultural, linguistic, and historical Lion Capital of the Ashoka Pillar in Sarnath, Uttar
differences among states, making it beneficial for Pradesh, was adopted as the State Emblem on
states with diverse religious, cultural, and ethnic January 26, 1950.

https://www.meritnest.com Page 40
Polity Level-2 Question Bank Historical Background, Constituent Assembly, Salient Features

53. The Supreme court of India can declare certain 54. Consider the following statements:
laws/executive actions as unconstitutional under
which of the following conditions? 1. Indian parliament is a sovereign body similar to
British parliament.
1. If the law enacted by parliament violates a
provision of the constitution. 2. Indian state has an elected head while the Britain
state has hereditary head.
2. Any laws that are against the federal provisions
of the constitution. 3. Political sovereignty in India is with the people
and not in a single individual.
3. Any legislative/executive actions that
contravenes the Directive Principles of State Policy How many statements given above are correct?
(DPSP).
(a) Only one
Select the correct answer using the code given (b) Only two
below: (c) All three
(a) 1 and 2 only (d) None
(b) 2 and 3 only
Solution 54. Correct Option: (b)
(c) 1 only
(d) 1 and 3 only Incorrect Statement: While the Indian
9654469135 Parliamentary
oslo System mirrors the British model,
significant differences exist. Key among these is the
Solution 53. Correct Option: (a)
sovereignty aspect, with the British Parliament
Judicial review empowers courts to assess the being sovereign, a status not shared by its Indian
constitutional validity of laws or executive actions. counterpart.
It's noteworthy that the constitution itself does not
explicitly mention "Judicial review." Correct Statement: India is considered a republic
due to its elected head of state, distinguishing it
Statement 1: Correct from the British system, which features a
hereditary monarchy.
Judicial review allows the Supreme Court to
evaluate the constitutionality of laws or executive Correct Statement: As a republic, India vests
actions. political power in its citizens, opposed to a monarch.
This ensures equal access to public offices for all
If such laws or actions breach constitutional
citizens, eliminating any form of privileged class.
provisions, the court can deem them
unconstitutional.
55. Consider the following statements:
Statement 2: Correct
1. Communist Party of India did not win any seats in
The Supreme Court can invalidate any laws or the elections to the Constituent Assembly of India.
actions that compromise the constitution's basic
structure. 2. Unionist Party won the second highest number of
seats, after Indian National Congress, in the
Given that federalism is a core aspect of the
elections to the Constituent Assembly of India.
constitution, any laws or actions undermining this
principle can be invalidated by the Supreme Court. katya124905@gmail.com
Which of the statements given above is/are
correct?
Statement 3: Incorrect
(a) 1 only
The Directive Principles of State Policy (DPSP) are (b) 2 only
non-justiciable, meaning they cannot be enforced
(c) Both 1 and 2
by the judiciary.
(d) Neither 1 nor 2
Therefore, the Supreme Court cannot void laws or
actions that conflict with the DPSP.
Solution 55. Correct Option: (d)

https://www.meritnest.com Page 41
Polity Level-2 Question Bank Historical Background, Constituent Assembly, Salient Features

The Constituent Assembly's members were


Charter Act of 1813 - Historical Context
selected by provincial assemblies through a single,
transferable-vote system, adhering to proportional
British merchants struggled due to Napoleon’s
representation.
Continental System, which blocked British imports
into France's European allies.
Correction to Statement 1: It erroneously states
the composition within the Assembly. In reality, the
They called for participation in Asia's trade and the
Indian National Congress dominated with 69% of
termination of the East India Company's monopoly,
seats. However, it also included representatives
facing opposition from the company.
from smaller parties like the Scheduled Caste
Federation, the Communist Party of India (which
The 1813 Act allowed British merchants to trade in
secured 1 seat), and the Unionist Party.
India through a regulated licensing system,
maintaining the company’s monopoly only in China
Correction to Statement 2: This statement katya124905@gmail.com
and tea trade.
inaccurately describes the assembly's party
distribution. The Muslim League was actually the
second-largest party with 73 seats, trailing behind 57. With reference to the ‘Objectives Resolution,
the Indian National Congress's 208 seats. The introduced in the first session of the Constituent
Unionist Party had a minimal presence with only 1 Assembly, consider the following statements:
seat.
1. It declared India as an Independent Sovereign
Republic.
56. With reference to the Charter Act of 1813,
oslo
consider the following statements:
2. It provided adequate safeguards for minorities
and depressed classes.
1. It granted permission to the Christian
9654469135
missionaries to come to India and engage in
3. It guaranteed to people of India, the equality of
religious proselytisation.
status of opportunity.
2. It extended the trade monopoly of the company
How many of the above given statements are
in India for another ten years.
correct?
3. It set aside Rs.1 Lakh every year on the education (a) Only one
of Indians. (b) Only two
(c) All three
How many statements given above are correct?
(d) None
(a) Only one
(b) Only two Solution 57. Correct Option: (c)
(c) All three
Jawaharlal Nehru introduced the Objectives
(d) None Resolution in the Constituent Assembly in
December 1946. It established the core principles
Solution 56. Correct Option: (b) and philosophy for the constitution's framework.
The Assembly unanimously adopted the Resolution
Statement 1: Correct. The Charter Act of 1813
on January 22, 1947, and its revised version now
allowed Christian missionaries to enter India for
constitutes the Preamble of India's current
religious proselytization.
Constitution.
Statement 2: Incorrect. The Charter Act of 1793,
Statement 1: The Objectives Resolution committed
not the 1813 Act, extended the East India
the Constituent Assembly to establish India as an
Company's trade monopoly for another 20 years.
Independent Sovereign Republic and to create a
The 1813 Act ended this monopoly except in tea
constitution that would guide the nation's
and China trade.
governance.
Statement 3: Correct. The Act of 1813 mandated
Statement 2: It ensured the provision of adequate
the Company to allocate Rs. 1 Lakh annually
safeguards for minorities, backward and tribal
towards Indian education.

https://www.meritnest.com Page 42
Polity Level-2 Question Bank Historical Background, Constituent Assembly, Salient Features

areas, as well as for depressed and other backward Statement 1: Incorrect. The Indian Councils Act of
classes. 1861 initiated decentralization by restoring
legislative powers to the Bombay and Madras
Statement 3: The Resolution pledged to provide all Presidencies, marking the start of representative
Indian citizens with justice—social, economic, and institutions through the inclusion of Indians in the
political; equality in status and opportunity; and law-making process. The Viceroy was allowed to
freedom of thought, expression, belief, faith, nominate some Indians as non-official members of
worship, vocation, association, and action, within his expanded council.
the bounds of law and public morality.
This act began the process of including Indians in
governance.
58. In the federation established by the
Government of India Act of 1935 residuary Power
Statement 2: Correct. The Indian Councils Act of
was given to the:
1892 expanded legislative council functions,
(a) Federal Legislature allowing discussions on the budget and enabling
(b) Governor-General questions to be asked to the executive.
(c) Provincial Legislature
This act enhanced the legislative councils' roles and
(d) Provincial Governors responsibilities.

Solution 58. Correct Option: (b) Statement 3: Correct. The Simon Commission's
1930 report recommended abolishing dyarchy,
The Government of India Act 1935 established an
9654469135 extending
osloresponsible government in provinces,
All-India Federation comprising British India
establishing a federation of British India with
Provinces and other Indian states, granting the
princely states, and continuing the communal
Governor General residuary powers. Specifically:
electorate, among other suggestions.
The Governor General had the authority to
Key recommendations included major governance
delegate legislation of any residuary matter to
reforms and federation establishment.
either the Federal or Provincial Legislature.

60. With reference to ‘Composition of Constituent


59. Consider the following statements with
Assembly of India, consider the following
reference to Acts and Commissions during the
statements:
British rule:
1. Each British Indian province and princely state
1. Indian Councils Act of 1833 provided for
were allotted fixed one seat in the constituent
nomination of Indians as non-official members of
assembly.
legislative council.
2. The seats allocated to each British province were
2. India Council Act of 1892 enlarged the functions
to be divided among the Muslims, Sikhs and
of the Legislative Councils and gave them powers to
General.
discuss the Budget.
3. The members from chief commissioners’
3. The Simon Commission recommended the
Provinces were represented in the constituent
extension of responsible Government in the
assembly.
provinces.
How many of the above given statements are
How many statements given above are correct? katya124905@gmail.com
correct?
(a) Only one
(a) Only one
(b) Only two
(b) Only two
(c) All three
(c) All three
(d) None
(d) None

Solution 59. Correct Option: (b)


Solution 60. Correct Option: (b)

https://www.meritnest.com Page 43
Polity Level-2 Question Bank Historical Background, Constituent Assembly, Salient Features

The Constituent Assembly was formed in options 1, 4, and 5.


November 1946, following the Cabinet Mission
Plan's guidelines. Borrowed provisions include:

Statement 1 (Incorrect): From the Australian Constitution:


Allocation of Seats: Provinces and princely states Concurrent List
were to receive seats based on their populations,
Freedom of trade, commerce, and intercourse
with an approximate distribution of one seat per
million people. Joint sitting of the two Houses of Parliament

Statement 2 (Correct): The amendment procedure is adapted from the


South African Constitution.
Division of Seats: In each British province, seats
were divided among Muslims, Sikhs, and the general
The concept of federation with a strong center is
population (everyone except Muslims and Sikhs).
inspired by the Canadian Constitution.
Community members in the provincial legislative
assemblies elected their representatives via
proportional representation using the single 62. Which among the following constitutional
transferable vote method. provision form the basis of India being a welfare
state?
Statement 3 (Correct):
1. Fundamental Rights
Chief Commissioner’s Provinces: These were
mid-level or minor Indian provinces post-colonial
2. Directive Principles of State Policy
period, led by a Chief Commissioner instead of a
(lieutenant-)governor. The Constituent Assembly
3. Fundamental Duties
had 389 members, with four from the Chief
Commissioners’ provinces, one from each province.
4. Preamble

61. Which of the following features of Indian Select the correct answer using the codes given
Constitution is/are borrowed from Australian below.
Constitution?
(a) 1, 2 and 4 only
1. Concurrent List (b) 1 and 4 only
(c) 1, 2 and 3 only
2. Procedure for amendment of the Constitution (d) 2 and 3 only
katya124905@gmail.com
3. Federation with a strong Centre
Solution 62. Correct Option: (a)
oslo
4. Joint sitting of the two Houses of Parliament Statement 1: Correct. Fundamental Rights aim to
eliminate discrimination and untouchability, and
5. Freedom of trade and commerce promote affirmative action for historically
marginalized groups such as Scheduled Castes,
Select the correct answer using the codes given Scheduled Tribes, and OBCs, thereby contributing
9654469135
below: to the establishment of a welfare state.
(a) 1, 2 and 4 only
Statement 2: Correct. The Directive Principles of
(b) 1, 4 and 5 only State Policy are designed to foster social and
(c) 2, 3 and 5 only economic democracy, aligning with the goal of
(d) 2, 3, 4 and 5 only creating a welfare state.

Statement 3: Incorrect. While Fundamental Duties


Solution 61. Correct Option: (b)
remind citizens of their responsibilities alongside
The Constitution of India incorporates elements their rights, they are not directly aimed at
from various international constitutions and the establishing a welfare state.
Government of India Act of 1935, reflecting a
diverse set of influences. Correct responses are

https://www.meritnest.com Page 44
Polity Level-2 Question Bank Historical Background, Constituent Assembly, Salient Features

Statement 4: Correct. The Preamble advocates for 64. Which of the following elements were
socio-economic and political justice, supporting the introduced for the first time in British India by the
vision of a welfare state. Government of India Act, 1919?

1. Dyarchy at provinces oslo


63. Provisions related to which of the following katya124905@gmail.com
topics of the Constitution of India came into force
2. Bicameralism
on 26th November 1949?
3. Communal representation in legislature
1. Provisional Parliament
4. Association of Indians with the Viceroy’s
2. Citizenship
executive council
3. Directive Principles of State Policy
5. Allowing Provincial Councils to determine on
women’s voting rights
4. Elections
Select the correct answer using the code given
5. Planning Commission
below:
6. Fundamental Rights (a) 1 and 2 only
(b) 1, 3 and 4 only
Select the correct answer using the codes given
(c) 1, 2 and 5 only
below:
(d) 2, 3, 4 and 5 only
(a) 1, 2, 3 and 5 only
(b) 1, 2 and 4 only Solution 64. Correct Option: (c)
(c) 2, 3, 4 and 6 only
Statement 1: Correct
(d) 1, 2, 3, 4, 5 and 6
The Government of India Act, 1919, reduced
central government control over provinces by
Solution 63. Correct Option: (b)
dividing and segregating central and provincial
Correct Options: subjects, thus introducing a dual governance
system known as dyarchy in British India's
Provisions related to citizenship, elections,
provinces.
provisional parliament, and temporary and
transitional provisions, as specified in Articles 5, 6,
Statement 2: Correct
7, 8, 9, 60, 324, 366, 367, 379, 380, 388, 391, 392,
9654469135
and 393, were implemented on November 26, The Act of 1919 introduced bicameralism and
1949. direct elections for the first time, replacing the
Indian Legislative Council with a bicameral
Incorrect Options: legislature consisting of an Upper House and a
Lower House.
The Directive Principles of State Policy and
Fundamental Rights provisions became effective on
Statement 3: Incorrect
January 26, 1950.
It was the Indian Councils Act of 1909, not the
The Planning Commission was not established by
Government of India Act of 1919, that initiated
the Constitution but rather as an
communal representation for Muslims through the
extra-constitutional entity.
concept of ‘separate electorate.’ The 1919 Act
expanded this by providing separate electorates for
Did You Know?
Sikhs, Indian Christians, Anglo-Indians, and
The choice of January 26, 1950, for India's Europeans.
Constitution to take effect was to mark the 20th
anniversary of the Lahore session of the Congress, Statement 4: Incorrect
led by Nehru. This date was selected to
The association of Indians with the executive
commemorate India's declaration of Independence
councils of the Viceroy and Governors was first
and to symbolize its transition to a republic.
enabled by the Indian Councils Act of 1909, not the
Government of India Act of 1919.

https://www.meritnest.com Page 45
Polity Level-2 Question Bank Historical Background, Constituent Assembly, Salient Features

enterprise, and social justice.


Statement 5: Correct
Error in Statement 3:
The Government of India Act 1919 allowed
Provincial Councils the discretion to enable The description of laissez-faire incorrectly merges
women's suffrage, contingent upon strict economic policies with the concept of a Police State.
qualifications regarding property, income, or Laissez-faire advocates for minimal government
education levels. intervention in economic affairs, aiming to ensure
freedom of transaction and protection against theft
and monopoly abuse, without endorsing a police
65. Which of the following statements is/are
state approach.
correct about the concept of ‘Police State’?
Interesting Fact:
1. It puts forward that the state can perform various
functions of social welfare. The laissez-faire principle has significantly
influenced state roles, transitioning from Police
2. It is concerned with the maintenance of law and State to Welfare State. This shift increased
order and defence of the country against external governmental responsibilities and emphasized the
aggression. importance of regulating and balancing
governmental power.
3. Laissez faire is the fundamental economic system
and policy necessarily followed by all Police states.
66. Why Indian judiciary has been described as an
integrated judiciary system?
Select the correct answer using the code given
below: (a) High court enforces only state laws of concerned
respective states.
(a) 1 and 2 only
(b) The laws of concurrent list are only enforced by
(b) 1 and 3 only
the Supreme court.
(c) 2 only
(c) Supreme Court has replaced the British Privy
(d) 1, 2 and 3 Council as the highest court: of appeal.
(d) The decisions made by higher courts are binding
Solution 65. Correct katya124905@gmail.com
Option: (c) on the lower courts.
The various perspectives have conceptualized the
state differently. Solution 66. Correct Option: (d)
The Indian Constitution establishes an integrated
Classical liberalism and the Police State
judicial system headed by the Supreme Court,
perspectives were prevalent in the 18th and early
followed by the High Courts, with a tier of
19th centuries,
oslo advocating for a state with minimal
subordinate courts (district and lower courts)
responsibilities.
beneath them. This structure ensures that decisions
9654469135 from higher courts are binding on the lower courts,
Correction of Statement 1:
weaving a unified thread from the Gram Panchayat
The assertion that Classical liberalism and the courts up to the Supreme Court.
Police State concept are synonymous is incorrect.
Classical liberalism, grounded in individualism, Integrated Judicial System:
views the state as a necessary but undesirable
India’s unified court system, based on the
entity due to its potential to infringe on individual
Government of India Act of 1935, enforces both
liberty. Conversely, the Welfare State, informed by
central and state laws through a single hierarchy of
revisionist theory, sees the state as an instrument
courts.
of social welfare, countering the negative view.
In contrast, the United States has a dual judicial
Statement 2 Clarification: system where federal courts enforce federal laws,
and state courts enforce state laws.
The Police State is characterized by a limited role of
the state, focusing on law enforcement, national
defense, justice, and taxation. This approach was 67. Consider the following statements:
later replaced by the Welfare State, which
emphasizes poverty alleviation, regulation of

https://www.meritnest.com Page 46
Polity Level-2 Question Bank Historical Background, Constituent Assembly, Salient Features

1. The original Constitution was handwritten in structure of Indian Constitution.


Hindi and English.
How many of the above given statements are
2. The 58th Constitutional Amendment Act correct?
provided for the translation of the Constitution in
(a) Only one
Hindi language to be published under the authority
of President. (b) Only two
(c) All three
Which of the statements given above is/are (d) None
correct?
(a) 1 only Solution 68. Correct Option: (c)
(b) 2 only Statement 1:
(c) Both 1 and 2
Correct. The separation of powers is a framework
(d) Neither 1 nor 2 where responsibilities, authorities, and powers are
distributed across different groups rather than
Solution 67. Correct Option: (c) centralized. It is fundamental in political systems,
distinguishing the legislative, executive, and judicial
The Constitution of India, initially adopted in
branches.
English by the Constituent Assembly, was also
officially translated into Hindi in 1950, following a
Statement 2:
resolution of the Assembly and under the
authorization9654469135
of the Constituent Assembly's Correct.oslo
This doctrine underpins the American
President. presidential system, ensuring the legislative,
executive, and judicial powers are vested in three
Statement 1: The original Constitution was independent entities.
handwritten in both Hindi and English, featuring
calligraphy by Prem Behari Narain Raizada. This Statement 3:
statement is accurate.
Correct. The Supreme Court recognized the
separation of powers among the legislature,
Statement 2: The 58th Constitution Amendment
executive, and judiciary as a core element of the
Act of 1987 introduced Article 394-A to Part XXII
Indian Constitution's structure.
of the Constitution, mandating the President to:

Publish the Constitution's translation into Hindi, 69. Which of the following advocated for universal
allowing necessary modifications for consistency adult franchise during pre-independence period in
with the language, style, and terminology used in India?
Hindi translations of Central Acts.
1. The Constitution of India Bill, 1895
Ensure the publication of Hindi translations for
every constitutional amendment made in English. 2. Nehru report, 1928
This statement is correct.
3. Government of India act, 1919
68. Consider the following statements with
4. Lucknow pact, 1916
reference to the concept of ‘Separation of
Powers’:
Select the correct answer using the code given
below: katya124905@gmail.com
1. It is an organisational structure where
responsibilities and powers are divided between (a) 3 and 4 only
groups rather than being centrally held. (b) 1, 2 and 4 only
(c) 2, 3 and 4 only
2. The doctrine of Separation of Powers is the basis
of the American Presidential system. (d) 1 and 2 only

3. Separation of Powers between the legislature, Solution 69. Correct Option: (d)
executive and judiciary is an element of basic

https://www.meritnest.com Page 47
Polity Level-2 Question Bank Historical Background, Constituent Assembly, Salient Features

Indian Freedom Movement and Universal Adult their legislative powers. It attempted to introduce
Franchise a system of open competition for selection of civil
servants.”
Central Demand:
Which one of the following acts is being described
A pivotal aspect of the Indian freedom movement
in the above given paragraph?
was the push for universal adult franchise. This
movement witnessed notable endorsements (a) Charter Act of 1833
through various historical documents and (b) Indian Councils Act of 1861
committees during the pre-independence era in
(c) Government of India Act of 1858
India.
(d) Government of India Act of 1919
Notable Advocacies:
Solution 70. Correct Option: (a)
Constitution of India Bill, 1895: Marking the katya124905@gmail.com
Charter Act of 1833 Key Features:
earliest effort advocating for universal adult
franchise, it proposed that every citizen should
Governor-General Role Centralized:
have the right to cast one vote for electing
members to both the Parliament of India and a local The act transformed the Governor-General of
Legislative Council. Bengal into the Governor-General of India,
consolidating all civil and military powers under this
Nehru Report, 1928: It specified that any person, position. Lord William Bentinck became the
regardless of sex, above the age of 21, and not inaugural holder of this role.
legally disqualified, should have the right to vote. oslo
This principle was echoed in subsequent proposals Legislative Powers Restructured:
such as the Gandhian Constitution of Free India
It stripped the governors of Bombay and Madras of
(1946) and Ambedkar’s States9654469135
and Minorities
their legislative powers, centralizing exclusive
(1945), emphasizing universal adult franchise.
legislative authority for all of British India with the
Governor-General of India.
Incorrect Assertions:
End of Commercial Ventures for the East India
Government of India Act, 1919: Contrary to
Company:
providing for adult suffrage, this act limited voting
rights to individuals meeting certain criteria like The Act signaled the cessation of the East India
property or land ownership, income, and municipal Company's commercial operations, turning it into a
tax payments. While it allowed provincial councils solely administrative entity. It declared that the
to decide on women's suffrage, it imposed strict company's territories in India were to be managed
property, income, or educational requirements. ‘in trust for His Majesty, His heirs, and successors’.

Lucknow Pact, 1916: Though it advocated for a Civil Service Recruitment Reform Attempt:
wide franchise, the pact, a collaborative effort
- Proposed to establish a system of open
between the Indian National Congress and the
competition for civil servant selection, aiming to
Muslim League, did not explicitly support universal
make the process more meritocratic. Nonetheless,
adult franchise.
this initiative was thwarted by opposition from the
Court of Directors.
Additional Insight:

Sapru Committee Report, 1945: Aimed at 71. Which of the following statements proves that
addressing minority issues, this report was a the authority of Indian Constitution is higher than
product of the Sapru Committee formed by the that of Parliament?
Non-Party Conference in November 1944. Its
(a) The Constitution makers were more eminent
proposals played a significant role in shaping
leaders than the Members of Parliament.
political and constitutional discussions in India.
(b) The Constitution of India was framed before the
Parliament came into existence.
70. “This act made the Governor-General of
(c) Indian Constitution provides for powers and
Bengal as the Governor-General of India. It
formation of the Parliament.
deprived the Governors of Bombay and Madras of

https://www.meritnest.com Page 48
Polity Level-2 Question Bank Historical Background, Constituent Assembly, Salient Features

(d) The Constitution gives the power to Parliament (c) Both 1 and 2
to alter its Basic Structure. (d) Neither 1 nor 2

Solution 71. Correct Option: (c) Solution 72. Correct Option: (d)
The Constitution of India, adopted by the During colonial rule, India was divided into British
Constituent Assembly on 26th November 1949 and Indian Provinces and Indian Princely States. The
enacted on 26th January 1950, was the result of Government of India Act, 1935, aimed to establish a
extensive deliberations to ensure a balanced "Federation of India," comprising both British India
distribution of power among the executive, and, optionally, the Princely States.
legislature, and judiciary. It delineates the
formation and authority of the Parliament, thereby Incorrect Statement 1:
establishing itself as the foundational source of
parliamentary power. Contrary to the statement, the Act allowed Princely
States to voluntarily join the Federation based on
Incorrect Reasoning about the Authority of the their rulers' decisions. In contrast, the Provinces
Indian Constitution: were automatically included.

Option A: Asserting that the Constitution's Incorrect Statement 2:


authority over Parliament is because its makers The statement incorrectly describes the Act's
were more eminent than Members of Parliament is provisions on authority. For Provinces, the
flawed. The prominence of the constituents does Federation had complete legislative and executive
not inherently elevate the Constitution's authority. authority over all federal matters. However, for
Indian States, this authority was subject to the
Option B: The fact that the Constitution was terms of their Accession Instruments.
created before Parliament does not directly infer its
superior authority. The sequential establishment
alone does not suffice as an explanation. 73. In July 1946, the Indian National Congress
appointed an ‘Experts Committee. With reference
Option D: The Basic Structure Doctrine, not original to this Experts Committee of Congress, consider
to the Constitution, aimed to restrict Parliament's the following statements:
amendment powers to protect the Constitution's
core principles. This limitation ensures the 1. It was appointed to prepare material for the
fundamental framework remains unaltered, but it Constituent Assembly.
was not a founding rationale for the Constitution's
authority over Parliament. 2. It recommended the draft of the Objectives
Resolution.

72. The Government of India Act of 1935 had 3. Krishna Kriplani was the chairman of this
provision of establishing an All-India Federation. committee.
In this context, how this act differentiated Indian
oslo
Princely States from the Governor's Provinces? How many statements given above are correct?

1. Indian Princely states were mandatorily needed (a) Only one


to join Indian federation whereas it was voluntary in (b) Only two
case of Governor's Provinces. (c) All three
9654469135
(d) None
2. Unlike Princely States, in case of Governor's —
Provinces the federal authority was extended to
katya124905@gmail.com
only legislative matters and not to executive Solution 73. Correct Option: (b)
matters. Before the Assembly elections concluded, the
Congress Party was already focused on the
Which of the statements given above is/are Constituent Assembly. On July 9, 1946, the
correct? Congress Working Committee established an
(a) 1 only Experts Committee for early constitution-drafting
efforts.
(b) 2 only

https://www.meritnest.com Page 49
Polity Level-2 Question Bank Historical Background, Constituent Assembly, Salient Features

Statement 1: Accurate. The Congress Party formed


Popular Sovereignty: It proclaimed that
an Experts Committee on July 8, 1946, during the
independent India would base its authority on the
Constituent Assembly elections, to prepare for the
people's power.
Constituent Assembly.
oslo
Federal Structure: The Resolution outlined a
katya124905@gmail.com
Statement 2: Correct. The committee
federal government structure for India, with a
recommended election processes for key Assembly
central authority and autonomous Provinces. It also
roles and the formation of vital committees. Its
suggested that residual powers should rest with the
most crucial contribution was the 'Declaration of
provinces to ensure their autonomy.
Objectives,' the precursor to the Objectives
Resolution.
Key Provisions of the Objectives Resolution:
Statement 3: Incorrect. Jawaharlal Nehru chaired
Sovereign Republic: Declared India as an
the Committee, and upon his suggestion, Krishna
independent Sovereign Republic.
Kriplani joined as a member and convener.
People's Power: Stated that all of India's sovereign
Additional Information:
power and its constitution would emanate from the
citizenry.
The Committee's members were Jawaharlal Nehru
(Chairman), K.M. Munshi, N Gopalsawmi Ayyangar,
Federal Government: Envisioned a federal
K.T. Shah, D.R. Gadgil, Humayun Kabir, and K
government system, detailing the division of
Santhanam. This composition shows the Congress
powers between the Centre and States.
Party's deliberate inclusion of non-members.
Justice and Equality: Guaranteed social, economic,
74. With reference to the ‘Objectives Resolution, and political justice; equality of status,
1946’, consider the following statements: opportunities, and before the law; along with
fundamental freedoms of speech, expression, belief,
1. It served as the inspiration for Preamble in the faith, worship, vocation, association, and action
constitution. within the bounds of law and public morality.

2. It envisioned Popular Sovereignty for the Protection for Minorities: Promised safeguards for
independent Indian nation. minorities, backward and tribal areas, and
depressed and other backward classes.
3. It provided for vesting of residuary powers with
the provinces.
75. Consider the following pairs regarding the
9654469135
various committees of the Constituent Assembly
How many of the above given statements are
and their Chairpersons:
correct?
(a) Only one 1. Union Powers Committe — Jawaharlal Nehru
(b) Only two
2. Provincial Constitution Committee — Dr. B.R.
(c) All three
Ambedkar
(d) None
3. Union Constitution Committee — Dr. Rajendra
Solution 74. Correct Option: (c) Prasad
Jawaharlal Nehru presented the Objectives
4. Minorities sub-committee — J.B. Kriplani
Resolution on December 13, 1946, which was
unanimously adopted on January 22, 1947. At this
How many pairs given above are correctly
point, the Muslim League had already boycotted the
matched?
Constituent Assembly, excluding themselves from
the discussions on the Resolution. (a) Only one
(b) Only two
Precursor to the Preamble: The Objectives
(c) Only three
Resolution inspired the Preamble of India's
Constitution. (d) All four

https://www.meritnest.com Page 50
Polity Level-2 Question Bank Historical Background, Constituent Assembly, Salient Features

Independence of the judiciary


Solution 75. Correct Option: (a)
Judicial review
The Constituent Assembly formed 22 committees
to devise the constitution, categorizing them into Impeachment of the President
eight major and the remainder as minor
Removal of Supreme Court and High Court judges
committees.
Office of the Vice President
Major Committees with Chairpersons:
Option B is correct; these aspects are from the
Union Powers Committee - Jawaharlal Nehru
Government of India Act, 1935:
Union Constitution Committee - Jawaharlal Nehru
Federal scheme
Provincial Constitution Committee - Sardar Patel
Office of the Governor
Drafting Committee - Dr. B.R. Ambedkar
Judiciary structure
Advisory Committee on Fundamental Rights,
Public Service Commissions
Minorities, and Tribal and Excluded Areas - Sardar
Patel, with five sub-committees: Emergency provisions
Fundamental Rights Sub-Committee - J.B. Kripalani Administrative details
Minorities Sub-Committee - H.C. Mukherjee
Option C is incorrect as it attributes features
North-East Frontier Tribal Areas and Assam borrowed from the Australian Constitution,
Excluded & Partially Excluded Areas including:
Sub-Committee - Gopinath Bardoloi
Concurrent List
Excluded and Partially Excluded Areas (other than
Freedom of trade, commerce, and intercourse
those in Assam) Sub-Committee - A.V. Thakkar
Joint sitting of the two Houses of Parliament
North-West Frontier Tribal Areas Sub-Committee
Rules of Procedure Committee - Dr. Rajendra Option D is incorrect; elements sourced from the
Prasad British Constitution include:
States Committee (Committee for Negotiating with Parliamentary government
States) - Jawaharlal Nehru
Rule of Law
Steering Committee - Dr. Rajendra Prasad
Legislative procedure
Single citizenship
76. Which of the following features of constitution
of India has been borrowed from Government of Cabinet system
India Act,1935?
Prerogative writs
(a) Fundamental rights, independence of judiciary, oslo
Parliamentary privileges
judicial review, impeachment of the president.
(b) Federal Scheme, Office of governor, Judiciary, Bicameralism
Public Service Commissions, Emergency provisions.
(c) Concurrent List, freedom of trade, commerce 77. Consider the following statements regarding
and inter-course, and joint sitting of the two Houses Federalism In India: 9654469135
of Parliament.
(d) Parliamentary government, Rule of Law, 1. The new economic policy of 1991 helped in
katya124905@gmail.com
legislative procedure, single citizenship, cabinet strengthening federalism in India.
system, prerogative writs.
2. Judicial Supremacy forms an essential part of
federal polity.
Solution 76. Correct Option: (b)
Option A is incorrect because the following 3. Indian model of federalism is “Coming together’,
elements were derived from the US Constitution: unlike US model which is “Holding together”.
Fundamental rights

https://www.meritnest.com Page 51
Polity Level-2 Question Bank Historical Background, Constituent Assembly, Salient Features

How many of the above given statements are India is considered a semi-federal state, also known
correct? as a quasi-federal state according to Prof. K.C.
Wheare. The Supreme Court of India acknowledges
(a) Only one
it as having a federal structure with a pronounced
(b) Only two central bias. This quasi-federal
oslo system is
katya124905@gmail.com
(c) All three characterized by an imbalanced power distribution
(d) None between the Central government and the states,
highlighting a prominent central authority.
Solution 77. Correct Option: (a)
Federal Features of the Indian Constitution:
A federal government divides powers between the
Constitution's supremacy.
national and regional governments, as laid out by
the Constitution, allowing each to operate Bicameral legislature.
independently within their jurisdictions.
Dual government.
Statement 1 (Correct): The 1991 economic reforms Rigid constitution.
bolstered federalism by enhancing fiscal federalism,
Independent judiciary.
marking a new direction for economic policy.
Federalism with a system of revenue sharing
Statement 2 (Incorrect): An independent judiciary between the Center and States.
exists to resolve disputes between the Central and
State governments, indicating judicial supremacy. Unitary Features of the Indian Constitution:
However, suggesting that judicial supremacy is
Single constitution.
neither essential nor desirable in a federal structure
is incorrect. The judiciary's role is critical in Rajya Sabha doesn't represent the States equally.
ensuring respect for its decisions and in guiding
Unequal division of powers (strong Center).
future public policy based on established legal
principles. Constitution is not entirely rigid, allowing
amendment by a simple majority in certain cases.
Statement 3 (Incorrect): The key difference
Integrated judiciary.
between "Coming Together" and "Holding
Together" federations is misconstrued. "Coming Emergency powers.
Together" federations involve independent states
uniting, while maintaining some power and identity,
79. With reference to the difference between East
to form a stronger nation. Conversely, in "Holding
India Company Rule and British Crown Rule in
Together" federations, a central government shares
India, consider the following statements:
9654469135
power with its constituent states to maintain unity.
India exemplifies the "Holding Together" model,
1. While local government did not have any
and the USA illustrates the "Coming Together"
financial powers under East India Company rule,
model, contrary to the incorrect assertion of power
they were given power to impose taxes under
sharing and nationality retention.
Crown rule.

78. Why India is sometimes referred to as a 2. While East India Company rule completely
‘quasi-federal’ polity? excluded Indians from civil services, Crown rule
paved the way for their recruitment in
(a) In India, executive is responsible to the
administration.
legislature for all its acts and policies.
(b) Indian constitution provides a Bicameral Which of the statements given above is/are
legislature for the Centre and the states. correct?
(c) India is a federal country with balance of power
(a) 1 only
tilted towards Centre.
(b) 2 only
(d) Indian constitution provides for division of
power between the Centre and the states. (c) Both 1 and 2
(d) Neither 1 nor 2
Solution 78. Correct Option: (c)
Solution 79. Correct Option: (d)

https://www.meritnest.com Page 52
Polity Level-2 Question Bank Historical Background, Constituent Assembly, Salient Features

Incorrect Statement 2: The Act led to the creation


In 1858, the British Parliament passed the
of the Supreme Court in Calcutta in 1774,
Government of India Act, transferring control of
consisting of a Chief Justice and three other judges.
India from the East India Company to the British
government.
Correct Statement 3: The Regulating Act of 1773
marked the British Parliament's initial effort to
Clarification on Statement 1: The notion that local
oversee the East India Company's operations,
governance was entirely exempt under East India
acknowledging its political and administrative roles
Company rule is inaccurate. For instance, the
for the first time.
Charter Act of 1813 allowed local governments in
India to levy taxes on individuals.
Did You Know?
Lord Ripon's Resolution of 1882 empowered The High Courts of Calcutta, Bombay, and Madras
Indians with rights to local self-governance, were founded under the Indian High Courts Act of
fostering the development of municipal institutions 1861, beginning their operations in 1862.
previously under British Crown control.
81. The main advantage of the parliamentary form
Clarification on Statement 2: Contrary to the
of governments is that:
second statement, the Charter Act of 1853
implemented an open competition system for civil (a) The executive and legislature work
service selection, thereby enabling Indian independently.
participation. Therefore, Indians were already being (b) It provides continuity of policy and is more
integrated into the administration under East India efficient.oslo
9654469135
Company's tenure.
(c) The executive remains responsible to the
legislature.
Post-1886, the civil service was rebranded as the
Imperial Civil Service. The Aitchison Commission of (d) The head of the government cannot be changed
1886 advocated for the inclusion of Indians in without election.
public service roles.
Solution 81. Correct Option: (c)
80. With reference to the Regulating Act of 1773, Option C is the accurate choice because the
consider the following statements: defining feature of a Parliamentary form of
government is the accountability of the executive
1. It made the governors of Bengal, Bombay and to the legislature regarding its policies and actions.
Madras presidencies independent of one another.
Ministers have a collective responsibility to
2. It provided for the establishment of High Courts Parliament, especially to the Lok Sabha, meaning
at Calcutta, Bombay and Madras. the Lok Sabha can dismiss the council of ministers,
including the Prime Minister, through a
3. It recognized, for the first time, the political and no-confidence motion.
administrative functions of the company. How
many statements given above are correct? Interesting Facts:
(a) Only one
Parliamentary vs. Presidential Systems:
(b) Only two
The parliamentary system fosters cooperation
(c) All three
between legislative and executive branches, while
(d) None the presidential system is characterized by a
katya124905@gmail.com
separation of powers.
Solution 80. Correct Option: (a)
A significant drawback of the parliamentary system
Incorrect Statement 1: The Regulating Act of 1773 is the potential lack of policy continuity, as the
designated the governors of Bombay and Madras uncertainty around government tenure can hinder
presidencies as subordinates to the the development and execution of long-term plans.
Governor-General of Bengal, altering their previous Policy changes often follow shifts in the ruling
independent status. party.

https://www.meritnest.com Page 53
Polity Level-2 Question Bank Historical Background, Constituent Assembly, Salient Features

Leadership can change without a general election in


the parliamentary system if the majority party's How many statements given above are correct?
Members of the Lok Sabha select a new leader, who
(a) Only one
then becomes the Prime Minister.
(b) Only two
(c) All three
82. ‘The people of India hold the power to freely
conduct the affairs of the nation-state. India is an (d) None
independent state and there is no authority above
it’. Solution 83. Correct Option: (b)
Statement 1: Correct
Which of the following term best defines this
attribute of India as a nation-state? The Amending Act of 1781, also known as the Act of
Settlement, addressed deficiencies in the
(a) Sovereignty
Regulating Act of 1773 by:
(b) Secularism
Exempting the Governor-General, the Council, and
(c) Republic
the company’s servants from the jurisdiction of the
(d) Democracy Supreme Court for actions performed in their
official capacity.
Solution 82. Correct Option: (a)
Statement 2: Correct
Sovereign
This Act further refined legal authority by:
Represents India as an independent entity, not
subject to any external control, ensuring its Excluding revenue matters and issues arising from
freedom to manage its own affairs. revenue collection from the Supreme Court's
jurisdiction.
Republic
Statement 3: Incorrect
Indicates that political power rests with the Indian
people, highlighting the absence of any privileged Contrary to the incorrect assertion, the Act indeed
class. stipulated:
The Supreme Court's jurisdiction extended to all
Secularism
Calcutta inhabitants.
Means the state does not endorse or favor any
The requirement for the court to apply personal
specific religion.
laws relevant to the defendant's religion: Hindu law
for Hindus, and Mohammedan law for Muslims.
Democracy
Signifies that government power is derived from
84. Which of the following statement(s) correctly
the people's consent. However, a democracy may
portray the secular features of Indian state as
not always be sovereign. For instance, while Hong
mentioned in the Constitution?
oslo
Kong is regarded as a democracy, it does not
possess sovereignty.
1. Freedom of transmission and dissemination of
one’s religious beliefs to others is guaranteed under
83. In the context of British rule in India, consider Indian Constitution.
the following statements regarding the Amending 9654469135
Act of 1781: 2. Indian state cannot spend the money collected by
way of tax for the promotion of any particular
katya124905@gmail.com
1. It exempted the servants of the company from religion.
the jurisdiction of the Supreme Court for their
official actions. 3. Constitution prohibits imparting of religious
instruction by any educational institutions in India.
2. It excluded the revenue matters from the
jurisdiction of the Supreme Court. Select the correct answer using the codes given
below:
3. It required the courts to administer a uniform law
(a) 1 only
for the people of all religions and communities.

https://www.meritnest.com Page 54
Polity Level-2 Question Bank Historical Background, Constituent Assembly, Salient Features

(b) 1 and 2 only


(c) 2 and 3 only Solution 85. Correct Option: (c)
(d) 1, 2 and 3 Incorrect Option A: Article 142 of the constitution
enables the apex court to issue necessary orders or
Solution 84. Correct Option: (b) decrees for complete justice in any pending case,
contradicting the claim.
The Constitution of India establishes the country as
a Secular State, meaning it does not recognize any
Incorrect Option B: The doctrine of Separation of
religion as the official state religion.
Power indeed fosters Division of Labor and
Functional Specialization, but these are not its core
Statement 1: Correct
principles within the constitution.
Article 25 guarantees individuals the right to
Correct Option C:
propagate their religious beliefs, which includes katya124905@gmail.com
sharing and explaining the principles of their faith The principle of Separation of Power establishes
to others. However, it explicitly excludes the right checks and balances among government organs.
to convert others to one's own religion, aligning
It aims to prevent government tyranny and ensure
with the broader principle that forcible conversions
justice without bias.
violate the "freedom of conscience" assured equally
to all individuals.
Incorrect Option D:
Statement 2: Correct The essence of Separation of Power lies in
facilitating checks and balances,
oslo
rather than solely
Article 27 stipulates that the State cannot use tax preventing power encroachment, which contradicts
revenues for the promotion or maintenance of any the assertion made.
specific religion. This ensures 9654469135
the State remains
impartial, avoiding favoritism, patronage, or Constitutional Provisions on Separation of Powers
support of one religion over others, and indicating
that public funds must not be utilized for the benefit Article 50: Encourages the state to distinguish the
of any single religion. judiciary from the executive.

Statement 3: Incorrect Articles 74 and 163: Limit court interference with


the Council of Ministers' advice to the President or
According to Article 28, no religious teachings are Governor.
permitted in educational institutions wholly funded
by the State. However, this prohibition does not Articles 122 and 212: Prevent courts from
apply to institutions that, while managed by the challenging the validity of Parliament and
State, were established through an endowment or Legislature proceedings.E89
trust that mandates religious instruction. This
distinction allows certain state-administered Articles 121 and 211: Prohibit Parliament and State
educational establishments to offer religious Legislature from discussing the conduct of Supreme
teachings based on their foundational Court and High Courts judges unless contemplating
requirements. removal proceedings.G82

85. What is the primary objective of including the 86. Which among the following provisions of the
feature of Separation of Power in the Constitution Constitution came into force on the date of
of India? adoption of the Constitution?
(a) To help the judiciary in providing complete
1. Superintendence, direction and control of
justice to the weaker sections of society.
elections to be vested in the Election Commission of
(b) To improve the division of Labor and Functional India.
specialisation of skills.
(c) To provide Checks and Balances to prevent the 2. Proclamation of the National Emergency.
misuse of power.
(d) To promote fast decision making by limiting the 3. Citizenship at the commencement of the
encroachment of powers between the branches. Constitution.

https://www.meritnest.com Page 55
Polity Level-2 Question Bank Historical Background, Constituent Assembly, Salient Features

Option A is incorrect:
4. There shall be a Council of Ministers with the
Prime Minister at the head to aid and advise the Cornwallis, not Wellesley, instituted and organized
President. the civil services in India.
He aimed to curtail corruption by:
Select the correct answer using the code given
below: - Increasing civil servants' salaries.
(a) 1, 3 and 4 only - Implementing promotions based on seniority.
(b) 1, 2 and 4 only
Option B is incorrect:
(c) 1 and 3 only
(d) 2, 3 and 4 only In 1800, Wellesley, not Cornwallis, established the
Fort William College for training new recruits.
Solution 86. Correct Option: (c) In 1806, the college faced disapproval from the
katya124905@gmail.com
court of directors, leading to the establishment of
Correct Statements:
the East India College at Haileybury in England.
Statement 1 & 3:
Option C is incorrect:
The Constitution was adopted on November 26,
The Lee Commission, not the Aitchison Committee,
1949. On this date, specific provisions related to
advocated for the immediate establishment of a
citizenship, elections, the provisional parliament,
Public Service Commission, as per the Government
temporary and transitional provisions, and the
of India Act, 1919.
short title became effective. These are outlined in oslo
Articles 5 (Citizenship at the commencement of the
Option D is correct:
Constitution), 6, 7, 8, 9, 60, 324 (Election
Commission of India), 366, 367, 379, 380, 388, 391,
9654469135 The Lee Commission (1924) suggested direct
392, and 393. recruitment to the Indian Civil Service with a 50:50
parity between Europeans and Indians to be
Incorrect Statements: achieved in 15 years.

Statement 2 & 4:
88. Which of the following statements regarding
The majority of the Constitution's provisions were the powers of the Legislature in British India is/
enacted on January 26, 1950, known as the are correct?
commencement date and celebrated as Republic
Day. Notable examples include Article 352 1. The Indian Councils Act of 1861 empowered the
(Proclamation of the National Emergency) and legislature to discuss and vote upon selected items
Article 74 (establishing a Council of Ministers of the budget for the first time.
headed by the Prime Minister to assist the
President), which took effect on this date. 2. The members were allowed to ask supplementary
questions for the first time by the Indian Councils
Act, 1909.
87. Which one of the following options is correct
with reference to Evolution of civil services in
3. The Government of India Act of 1919 gave the
India?
legislators the right to move adjournment motions
(a) Wellesley was the first to bring into existence and raise urgent issues of public interest.
and organize the civil services in India.
(b) Cornwallis set up the Fort William college for Select the correct option using the code given
training of new recruits. below:
(c) Aitchison committee recommended for a Public (a) 1 and 2 only
Service Commission to be immediately established. (b) 2 and 3 only
(d) Lee commission recommended 50:50 parity (c) 1 and 3 only
between the Europeans and the Indians in direct
(d) 1, 2 and 3
recruitment of ICS.

Solution 88. Correct Option: (b)


Solution 87. Correct Option: (d)

https://www.meritnest.com Page 56
Polity Level-2 Question Bank Historical Background, Constituent Assembly, Salient Features

Statement 1 is Inaccurate: The formation of a These principles guide the Indian government in
legislative council in British India under the Indian enacting laws that aim to reduce poverty.
Councils Act, 1861, marked an initial effort to
introduce a legislative body. However, its abilities Statement 2: Correct.
were highly restricted, with no provision to
The Constitution establishes fundamental rules for
challenge or vote on executive or financial issues.
societal coordination.
Statement 2 is Accurate: The Indian Councils Act of It ensures order to a minimal degree, while societal
1909, also known as the Morley-Minto Reforms, harmony depends on individuals' interactions.
significantly enhanced the legislative powers within
British India. For the first time, members could Statement 3: Correct.
interpellate and ask supplementary questions to
Laws must comply with the Constitution, the
hold the executive accountable for their actions.
supreme law of the land.
Statement 3 is Accurate: The Government of India If a law contravenes the Constitution, the judiciary
Act, 1919, introduced by the can invalidate it, ensuring laws' legality.
Montague-Chelmsford reforms, greatly
empowered the legislatures. This marked a Statement 4: Incorrect.
dramatic shift from their initial limitations, allowing
The Constitution outlines governance procedures
members to propose adjournment motions. This
and principles, not who governs.
tool enabled them to interrupt regular proceedings
to address urgent public interest issues, thereby It does not address the personal values of elected
enabling a mechanism for the censure of the officials; this falls under constitutional morality.
executive.
As B.R. Ambedkar noted, the effectiveness of a
constitution depends on its executors.
89. Which of the following functions are served by
the Constitution of India?
90. Which of the following statements is/are
correct about the Constitution of India?
1. It serves as guide to the state to institute laws
and policies to reduce poverty.
1. It sets limits on the power of the government.
katya124905@gmail.com
2. It provides a set of basic rules for coordination
2. It provides an enabling framework for the
amongst members of a society.
government to take steps for fulfilling the
aspirations and goals of society.
3. It helps judiciary to decide the legality of the laws
framed by the legislatures.
Select the correct answer using the code given
oslo below:
4. It helps in ensuring that people with good morals
and values reach to power. (a) 1 only
9654469135
(b) 2 only
Select the correct answer using the code given
(c) Both 1 and 2
below:
(d) Neither 1 nor 2
(a) 1, 3 and 4 only
(b) 1 and 3 only Solution 90. Correct Option: (c)
(c) 2, 3 and 4 only
Statement 1: Governmental Limits Set by the
(d) 1, 2 and 3 only Constitution

Solution 89. Correct Option: (d) The Constitution establishes clear boundaries for
governmental actions, ensuring that certain limits
Statement 1: Correct.
are never to be overstepped. It specifies
The Constitution of India includes the Directive fundamental rights that are intrinsic to all citizens,
Principles of State Policy to promote social and thereby restricting government power to ensure
economic reform. the protection of these rights.

https://www.meritnest.com Page 57
Polity Level-2 Question Bank Historical Background, Constituent Assembly, Salient Features

Statement 2: The Constitution's Role in Promoting reserved posts. However, civil servants appointed
Positive Government Actions before August 15, 1947, retained their pre-existing
benefits.
The Constitution not only restrains the government
but also empowers it to pursue societal goals and Correct Statement: It abolished the office of the
aspirations, such as alleviating inequality and Secretary of State for India, transferring its
eliminating caste discrimination. This is particularly responsibilities to the Secretary of State for
relevant in contexts like India, where the Commonwealth Affairs.
Constitution mandates the government to act
towards ensuring minimal dignity and social
92. With reference to the Pitts India Act of 1784,
self-respect for every individual. This includes
consider the following:
provisions for material well-being and education,
among other welfare measures, many of which are
1. It created the Board of Control to manage the
enforceable by law.
commercial affairs. katya124905@gmail.com

91. Consider the following statements regarding 2. It empowered the Court of Directors to supervise
Indian Independence Act of 1947: and direct all operations of the civil and military
government.
1. It deprived the Viceroy of India of legislative
powers and retained the office merely as a Which of the statements given above is/are
ceremonial position. correct?
(a) 1 only
2. It allowed the princely states either to join India oslo
or Pakistan but denied the right to remain (b) 2 only
independent. (c) Both 1 and 2
9654469135
(d) Neither 1 nor 2
3. It discontinued the appointment to civil services
by the secretary of state for India.
Solution 92. Correct Option: (d)
4. It abolished the office of secretary of state for Statement 1 Correction:
India.
Incorrect: The Pitt's India Act did not permit the
Court of Directors to manage commercial affairs.
How many statements given above are correct?
Correction: It established the Board of Control to
(a) Only one
oversee political affairs, thus creating a division of
(b) Only two responsibilities.
(c) Only three
(d) All four Statement 2 Correction:
Incorrect: The act did not solely empower the
Solution 91. Correct Option: (b) Board of Control to supervise civil and military
operations or revenue matters in British India.
Incorrect Statement: The Indian Independence Act
of 1947 eliminated all powers of the British Correction: It clarified the separation of
Monarch over legislation and transferred legislative commercial and political duties within the
authority to the respective constituent assemblies. Company.
It replaced the Viceroy of India with a
Governor-General for each new dominion.
93. With reference to the system of checks and
balances in the constitution, which of the
Incorrect Statement: The Act declared an end to
following statements is correct?
British control over Indian princely states and tribal
areas from August 15, 1947, allowing them to join (a) It enables the legislature to hold more power
either India or Pakistan or remain independent. over the executive and the judiciary.
(b) It is antithetical to the concept of division of
Correct Statement: The 1947 Act stopped the powers in a political structure.
Secretary of State for India from making
(c) Article 13 is an example of the system of checks
appointments to civil services and maintaining
and balances in India.

https://www.meritnest.com Page 58
Polity Level-2 Question Bank Historical Background, Constituent Assembly, Salient Features

(d) Single party majority in the parliament head.


strengthens the system of checks and balances.
Statement 2: Incorrect - Although the Viceroy
headed the Council, Jawaharlal Nehru held the
Solution 93. Correct Option: (c)
position of Vice-President within the Council.
Option C is Correct
95. With reference to the comparison between the
Article 13 exemplifies India's checks and balances
Government of India Act, 1919 and the
system, showcasing judicial review where the
Government of India Act, 1935, consider the
judiciary can invalidate laws passed by the
following statements:
parliament if they violate the constitution.
1. Both the Acts provided for the establishment of
Did You Know?
Dyarchy, but at different levels of federal polity.
katya124905@gmail.com
Checks and Balances System: This system allows
2. While the Act of 1919 provided for indirect
each of the three government branches to limit the
elections, the Act of 1935 provided for direct
powers of the others, preventing any single branch
elections to the central legislature.
from becoming overly powerful.
3. While the Act of 1919 introduced Bicameralism
Separation vs. Division of Powers: The system of
at the Centre, the Act of 1935 introduced it in
checks and balances contrasts with the separation
selected Provinces as well.
of powers, which distributes authority among
different state organs (executive, legislature,
How many of the above given
oslo statements are
judiciary). In contrast, the division of powers refers
correct?
to power distribution among different government
levels (central, state, local). (a) Only one
9654469135
(b) Only two
Impact of Single Party Majority: A single party
(c) All three
majority in parliament can undermine the checks
and balances system, as it leads to both the (d) None
executive and legislature being dominated by one
political entity. Solution 95. Correct Option: (b)
Correct Statements:
94. With reference to the interim government
formed in 1946, consider the following Government of India Act of 1919:
statements:
Introduced Dyarchy at the Provincial level.
1. The members of the interim Government were Introduced Bicameralism at the Centre, marking its
members of the Viceroy’s Executive Council. first adoption.

2. Jawaharlal Nehru was designated as the Government of India Act of 1935:


President of the Council.
Extended Dyarchy to the Central level.
Which of the statements given above is/are Continued the practice of Bicameralism at the
incorrect? Centre and introduced it in selected provinces
(Bengal, Bombay, Madras, Bihar, Assam, United
(a) 1 only
Provinces).
(b) 2 only
(c) Both 1 and 2 Incorrect Statement:
(d) Neither 1 nor 2 Both the 1919 and 1935 Acts facilitated Direct
Elections at both the Centre and Provincial levels,
Solution 94. Correct Option: (b) contrary to the statement that only the 1935 Act
did so.
Statement 1: Correct - The interim Government's
members were also part of the Viceroy's Executive
Council, with the Viceroy serving as the Council's

https://www.meritnest.com Page 59

You might also like